Крок 1 - Медицина 2009 (буклет)

1 / 200
Хворий 30-ти років звернувся до лікаря зі скаргами на пронос і біль у животі протягом 5-ти днів, підвищення температури тіла до 37,5oC з ознобами. Напередодні хворий був у лісі, де випив води з відкритого водоймища. Встановлено лабораторно підтверджений діагноз: амебна дизентерія. Вкажіть препарат вибору для лікування цього захворювання: A 30-year-old patient consulted a doctor with complaints of diarrhea and abdominal pain for 5 days, an increase in body temperature to 37.5oC with chills. The day before the patient was in the forest, where he drank water from an open reservoir. A laboratory-confirmed diagnosis was established: amoebic dysentery. Specify the drug of choice for the treatment of this disease:

Фталазол Phthalazole

Левоміцетин Levomycetin

Еметина гідрохлорид Emetine hydrochloride

Фуразолідон Furazolidone

Метронідазол Metronidazole

2 / 200
Чоловік, що страждає на спадкову хворобу, одружився із здоровою жінкою. У них було 5 дітей, три дівчинки і два хлопчика. Усі дівчатка успадкували хворобу батька. Який тип спадкування цього захворювання? A man suffering from a hereditary disease married a healthy woman. They had 5 children, three girls and two boys. All the girls inherited the disease from their father. What type inheritance of this disease?

Зчеплений з Y-хромосомою Y-linked

Аутосомно-домінантний Autosomal dominant

Рецесивний, зчеплений з X -хромосомою X-linked recessive

Домінантний, зчеплений з X -хромосомою Dominant, X-linked

Аутосомно-рецесивний Autosomal recessive

3 / 200
На електронній мікрофотографії представлена клітина макрофагічної природи, вздовж відростків якої розташовуються еритроцити на різних стадіях диференціювання. Клітина якого органу представлена? The electron micrograph shows a cell of a macrophage nature, along the processes of which erythrocytes are located at different stages of differentiation. The cell of which organ is represented?

Мигдалик Amygdalik

Тимус Thymus

Селезінка Spleen

Червоний кістковий мозок Red bone marrow

Лімфатичний вузол Lymph node

4 / 200
Досить часто причиною набутих імунодефіцитів є інфекційне ураження організму, при якому збудники безпосередньо розмножуються в клітинах імунної системи і руйнують їх. Виберіть серед перерахованих ті захворювання, при яких має місце вищезгадане: Quite often the cause of acquired immunodeficiency is an infectious lesion of the body, in which pathogens multiply directly in the cells of the immune system and destroy them. Choose from the listed diseases in which above:

Туберкульоз, мікобактеріоз Tuberculosis, mycobacteriosis

Поліомієліт, гепатит А Polio, hepatitis A

Ку-гарячка, висипний тиф Ku fever, typhus

Дизентерія, холера Dysentery, cholera

Інфекційний мононуклеоз, СНІД Infectious mononucleosis, AIDS

5 / 200
В шкірі виявлена щільна, рухома, чітко відмежована від оточуючих тканин пухлина. На розрізі вона білого кольору, представлена волокнистою тканиною. Мікроскопічно: хаотично переплетені колагенові волокна, клітин мало. Що це за пухлина? A dense, mobile tumor, clearly separated from the surrounding tissues, was detected in the skin. On cross-section, it is white in color, represented by fibrous tissue. Microscopically: chaotically intertwined collagen fibers, few cells . What kind of tumor is this?

Гістіоцитома Histiocytoma

Дерматофіброма Dermatofibroma

Міома Myoma

Фіброма Fibroma

Десмоїд Desmoid

6 / 200
У хворого після оперативного видалення кісти підшлункової залози виник геморагічний синдром з вираженим порушенням зсідання крові. Розвиток цього ускладнення пояснюється: After the surgical removal of a pancreatic cyst, the patient developed a hemorrhagic syndrome with a marked violation of blood coagulation. The development of this complication is explained by:

Активацією протизгортальної системи By activating the anti-collision system

Недостатнім утворенням фібрину Insufficient fibrin formation

Активацією фібринолітичної системи Activation of the fibrinolytic system

Зменшенням кількості тромбоцитів Decreased platelet count

Активацією фактору Крисмаса By activating the Christmas factor

7 / 200
Мати звернулася до лікаря з приводу того, що у дитини 5-ти років під дією сонячних променів на шкірі з’являються еритеми, везикулярний висип, свербіж шкіри. Лабораторні дослідження виявили зменшення вмісту заліза у сироватці крові, збільшення виділення з сечею уропорфіриногену І. Найбільш вірогідною спадковою патологією у дитини є: The mother consulted a doctor about the fact that a 5-year-old child develops erythema, a vesicular rash, and itchy skin under the influence of sunlight. Laboratory studies have revealed a decrease in the iron content in the blood serum, an increase in the excretion of uroporphyrinogen I in the urine. The most likely hereditary pathology in the child is:

Копропорфірія Coproporphyria

Печінкова порфірія Hepatic porphyria

Метгемоглобінемія Methemoglobinemia

Інтермітуюча порфірія Intermittent porphyria

Еритропоетична порфірія Erythropoietic porphyria

8 / 200
Немовля відмовляється від годування груддю, збудливе, дихання неритмічне, сеча має специфічний запах 'пивної закваски' або 'кленового сиропу'. Вроджений дефект якого ферменту викликав дану патологію? The baby refuses to breastfeed, is excitable, breathing is irregular, urine has a specific smell of 'beer sourdough' or 'maple syrup'. The congenital defect of which enzyme caused this pathology?

УДФ-глюкуронілтрансфераза UDF-glucuronyltransferase

Дегідрогеназа розгалужених альфа-кетокислот Dehydrogenase of branched alpha-keto acids

Глюкозо-6-фосфатдегідрогеназа Glucose-6-phosphate dehydrogenase

Гліцеролкіназа Glycerolkinase

Аспартатамінотрансфераза Aspartate aminotransferase

9 / 200
У мікропрепараті, виготовленому з пунктату регіонарного лімфовузла хворого, зафарбованому за Романовським-Гімзою, лікар виявив тонкі мікроорганізми з 12-14 рівномірними завитками з гострими кінцями, довжиною 10-13 мкм блідо-рожевого кольору. Про збудника якої інфекційної хвороби може йти мова в даному випадку? In a micropreparation made from a punctate of the patient's regional lymph node, stained according to Romanovsky-Giemsa, the doctor found thin microorganisms with 12-14 uniform curls with sharp ends, 10- 13 microns of pale pink color. What infectious disease agent can we be talking about in this case?

Сифіліс Syphilis

Лептоспіроз Leptospirosis

Лейшманіоз Leishmaniasis

Трипаносомоз Trypanosomiasis

Поворотний тиф Typhoid

10 / 200
При санітарно-бактеріологічному дослідженні води методом мембранних фільтрів виявлено дві червоні колонії на мембранному фільтрі (середовище Ендо), через який пропустили 500 мл досліджуваної води. Розрахуйте колі-індекс та колі-титр досліджуваної води: During the sanitary-bacteriological examination of water using the membrane filter method, two red colonies were detected on the membrane filter (Endo's medium), through which 500 ml of the tested water was passed. Calculate the coli index and the coli titer of the tested water:

2 та 500 2 and 500

250 та 4 250 and 4

250 та 2 250 and 2

4 та 250 4 and 250

500 та 2 500 and 2

11 / 200
Лікар-отоларинголог при огляді хворого відмітив гіперемію, значний набряк мигдаликів з сірим нальотом на них. При мікроскопії нальоту було виявлено грам-позитивні палички, розташовані під кутом одна до одної. Про яке захворювання слід думати? During the examination of the patient, the otolaryngologist noted hyperemia, significant swelling of the tonsils with a gray plaque on them. Microscopy of the plaque revealed gram-positive rods located at an angle to one. What disease should we think about?

Скарлатина Scarlatina

Дифтерія Diphtheria

Ангіна Angina

Епідемічний паротит Epidemic mumps

Менінгоназофарингіт Meningonasopharyngitis

12 / 200
До приймального відділення доставлено жінку 38-ми років з матковою кровотечею, що триває другу добу. Що з наведеного буде виявлено при аналізі крові хворої? A 38-year-old woman was brought to the reception department with uterine bleeding that has been going on for two days. Which of the following will be detected during the patient's blood analysis?

Уповільнення ШОЕ ESR slowdown

Лейкоцитоз Leukocytosis

Збільшення колірного показника Increasing color index

Еозинофілія Eosinophilia

Зменшення гематокритного показника Hematocrit decrease

13 / 200
У чоловіка після гіпертонічної кризи відзначається відсутність довільних рухів в правих руці та нозі, тонус м’язів у цих кінцівках підвищений. Який вид розладу рухової функції спостерігається у даному випадку? In a man, after a hypertensive crisis, the absence of voluntary movements in the right arm and leg is noted, the muscle tone in these limbs is increased. What type of motor function disorder is observed in this case ?

Периферичний парез Peripheral paresis

Периферичний параліч Peripheral paralysis

Рефлекторний парез Reflex paresis

Центральний параліч Central paralysis

Центральний парез Central paresis

14 / 200
У хворого в організмі знижений вміст іонів магнію, які потрібні для прикріплення рибосом до гранулярної ендоплазматичної сітки. Відомо, що це призводить до порушення біосинтезу білка. Який саме етап біосинтезу білка буде порушено? The patient has a reduced content of magnesium ions in the body, which are required for the attachment of ribosomes to the granular endoplasmic reticulum. It is known that this leads to a violation of protein biosynthesis. What exactly is the stage of biosynthesis protein will be disturbed?

Активація амінокислот Activation of amino acids

Трансляція Broadcast

Транскрипція Transcription

Термінація Termination

Реплікація Replication

15 / 200
Хворий з виразковою хворобою шлунка приймав антацидний препарат альмагель. Для лікування гострого бронхіту йому призначили антибіотик метициклін. Проте протягом 5- ти днів температура не знизилася, кашель і характер харкотиння не змінились. Лікар прийшов до висновку про несумісність ліків при їх взаємодії. Про який саме вид несумісності ліків йдеться? A patient with a stomach ulcer took the antacid drug Almagel. To treat acute bronchitis, he was prescribed the antibiotic methicycline. However, for 5 days the temperature did not decrease, the cough and the nature of sputum have not changed. The doctor came to the conclusion about the incompatibility of the drugs due to their interaction. What kind of incompatibility of the drugs are we talking about?

Фармакодинамічна Pharmacodynamic

Фармакокінетична на етапі біотрансформації Pharmacokinetic at the stage of biotransformation

Фармакокінетична на етапі всмоктування Pharmacokinetic at the stage of absorption

Фармацевтична Pharmaceutical

Прямий антагонізм Direct antagonism

16 / 200
У хворого 70-ти років атеросклероз ускладнився тромбозом судин нижніх кінцівок, виникла гангрена пальців лівої стопи. Початок тромбоутворення, найбільш вірогідно, пов’язаний з: In a 70-year-old patient, atherosclerosis was complicated by thrombosis of the vessels of the lower extremities, gangrene of the toes of the left foot occurred. The onset of thrombus formation is most likely associated with:

Активацією протромбінази Prothrombinase activation

Перетворенням фібриногену в фібрин Conversion of fibrinogen into fibrin

Перетворенням протромбіну в тромбін Conversion of prothrombin to thrombin

Зниженням синтезу гепарину Reduction of heparin synthesis

Адгезією тромбоцитів By platelet adhesion

17 / 200
У хворого 44-х років на ЕКГ виявлені ознаки гіпертрофії обох шлуночків та правого передсердя. Діагностовано недостатність тристулкового клапана. Який патогенетичний варіант порушення функції серця має місце при цій недостатності? In a 44-year-old patient, the ECG revealed signs of hypertrophy of both ventricles and the right atrium. Tricuspid valve insufficiency was diagnosed. What pathogenetic variant of heart dysfunction occurs with this insufficiency ?

Первинна міокардіальна недостатність Primary myocardial insufficiency

Тампонада серця Cardiac tamponade

Перевантаження серця об’ємом Heart volume overload

Перевантаження серця опором Heart overload with resistance

Коронарна недостатність Coronary insufficiency

18 / 200
Бактеріологічне дослідження гнійних виділень з уретри виявило наявність бактерій, які за Грамом фарбувалися негативно, нагадували кавові зернини, розкладали глюкозу і мальтозу до кислоти, розташовувалися в лейкоцитах. Збудники якої хвороби виявлені? Bacteriological examination of purulent secretions from the urethra revealed the presence of bacteria that stained negatively on Gram, resembled coffee beans, decomposed glucose and maltose into acid, were located in leukocytes. The causative agents of diseases detected?

М ’який шанкр Mild chancre

Венеричний лімфогранулематоз Venereal lymphogranulomatosis

Сифілісу Syphilis

Меліоїдоз Melioidosis

Гонорея Gonorrhea

19 / 200
Прозерин при системному введенні підвищує тонус скелетних м’язів. Фторотан викликає релаксацію м’язів і послаблює ефекти прозерину. Який характер взаємодії прозерину та фторотану? Proserin, when administered systemically, increases the tone of skeletal muscles. Fluorotane causes muscle relaxation and weakens the effects of proserin. What is the nature of the interaction between proserin and fluorotane?

Конкурентний антагонізм Competitive antagonism

Неконкурентний антагонізм Non-competitive antagonism

Непрямий функціональний антагонізм Indirect functional antagonism

Незалежний антагонізм Independent antagonism

Прямий функціональний антагонізм Direct functional antagonism

20 / 200
У жінки 62-х років розвинулася катаракта (помутніння кришталика) на фоні цукрового діабету. Який тип модифікації білків має місце при діабетичній катаракті? A 62-year-old woman developed a cataract (clouding of the lens) on the background of diabetes. What type of protein modification occurs in a diabetic cataract?

АДФ-рибозилювання ADP-ribosylation

Фосфорилювання Phosphorylation

Метилювання Methylation

Обмежений протеоліз Limited proteolysis

Глікозилювання Glycosylation

21 / 200
У дитини 2-х років виявлено відставання у фізичному розвитку, часті пневмонії. Встановлено діагноз - незарощення артеріальної протоки. Сполучення яких кровоносних судин викликало порушення гемодинаміки? A 2-year-old child was found to be retarded in physical development, frequent pneumonia. A diagnosis of non-union of the ductus arteriosus was made. The combination of which blood vessels caused a hemodynamic disorder?

Легеневий стовбур і легеневі вени Pulmonary trunk and pulmonary veins

Верхня порожниста вена і аорта Superior vena cava and aorta

Верхня порожниста вена і легеневий стовбур Superior vena cava and pulmonary trunk

Аорта і легеневі вени Aorta and pulmonary veins

Аорта і легеневий стовбур Aorta and pulmonary trunk

22 / 200
в середовищі з температурою 38o 22. C та відносною вологістю повітря 50%. Які шляхи тепловіддачі зумовлюють підтримку постійної температури ядра тіла за цих умов? in an environment with a temperature of 38o 22.C and a relative humidity of 50%. What heat transfer pathways cause the maintenance of a constant body core temperature under these conditions?

Конвекція і теплопроведення Convection and heat conduction

Радіація Radiation

Конвекція Convection

Випаровування Evaporation

Теплопроведення Heat conduction

23 / 200
До медико-генетичної консультації звернулося подружжя з питанням про вірогідність народження у них дітей, хворих на гемофілію. Подружжя здорове, але батько дружини хворий на гемофілію. В цій родині на гемофілію можуть захворіти: A couple turned to medical and genetic counseling with the question of the probability of having children with hemophilia. The couple is healthy, but the wife's father is hemophiliac. In this family can get hemophilia:

Половина дочок Half daughters

Половина синів Half sons

Тільки дочки Only daughters

Всі діти All children

Сини та дочки Sons and Daughters

24 / 200
У хворого з пересадженим серцем при фізичному навантаженні збільшився хвилинний об’єм крові. Який механізм регуляції забезпечує ці зміни? The minute volume of blood increased during physical exertion in a patient with a transplanted heart. What regulation mechanism ensures these changes?

Симпатичні безумовні рефлекси Sympathetic unconditioned reflexes

Катехоламіни Catecholamines

Симпатичні умовні рефлекси Sympathetic conditioned reflexes

Парасимпатичні безумовні рефлекси Parasympathetic unconditioned reflexes

Парасимпатичні умовні рефлекси Parasympathetic conditioned reflexes

25 / 200
На прийом до лікаря прийшла хвора зі скаргами на розлад травлення, розлитий біль у животі. При обстеженні виявлено різке зниження вмісту гемоглобіну в крові. З анамнезу відомо, що, під час перебування на Далекому Сході, вона часто вживала в їжу малосольну риб’ячу ікру. Аналогічний стан відзначений у деяких родичів, що проживають з нею. Яке захворювання найбільш вірогідне? A patient came to see a doctor with complaints of indigestion, diffuse pain in the stomach. During the examination, a sharp decrease in the hemoglobin content in the blood was detected. From the anamnesis, it is known that, during her stay in the Far East, she often ate low-salt fish roe. A similar condition was noted in some relatives living with her. What disease is most likely?

Трихінельоз Trichinellosis

Ехінококоз Echinococcosis

Теніоз Taeniosis

Дифілоботріоз Diphyllobotriosis

Аскаридоз Ascariasis

26 / 200
У людини нормальна чутливість шкіри пальця, але він не відчуває наявності на ньому обручки. Який процес, спричинений впливом обручки, є причиною цього? A person has normal finger skin sensitivity, but does not feel the presence of a wedding ring on it. What process caused by the influence of the wedding ring is the reason for this?

Порушення структури рецепторів Violation of receptor structure

Адаптація рецепторів Adaptation of receptors

Порушення структури епідермісу Violation of the structure of the epidermis

Розвиток фіброзної тканини Development of fibrous tissue

Порушення кровообігу Blood circulation disorder

27 / 200
У хворого 45-ти років з підозрою на запалення оболонок мозку потрібно було отримати спинномозкову рідину Зроблено діагностичну пункцію між дугами поперекових хребців (L3 — S4).Через яку зв’язку пройшла голка під час пункції? A 45-year-old patient with suspected inflammation of the meninges needed to obtain cerebrospinal fluid. A diagnostic puncture was made between the arches of the lumbar vertebrae (L3 — S4). Through which 'Did the needle go through the needle during the puncture?

Жовта Yellow

Передня поздовжня Front Longitudinal

Міжпоперечна Intertransverse

Клубово-поперекова Iliolumbar

Задня поздовжня Back Longitudinal

28 / 200
При аналізі ЕКГ людини з’ясовано, що у стандартних відведеннях від кінцівок зубці T позитивні, їх амплітуда та тривалість нормальні. Вірним є висновок, що у шлуночках серця нормально відбувається такий процес: When analyzing a human ECG, it was found that in the standard leads from the limbs, T waves are positive, their amplitude and duration are normal. The correct conclusion is that the ventricles of the heart are normal the following process takes place:

Скорочення Abbreviation

Реполяризація Repolarization

Розслаблення Relax

Збудження Arousal

Деполяризація Depolarization

29 / 200
У хворого в крові збільшена концентрація пірувату. Значна його кількість екскретується з сечею. Дефіцит якого вітаміну має місце у хворого? The patient has an increased concentration of pyruvate in the blood. A significant amount of it is excreted in the urine. What vitamin deficiency does the patient have?

E E

B1 B1

В3 B3

В2 B2

B6 B6

30 / 200
Людина захворіла на пелагру. При опитуванні стало відомо, що впродовж тривалого часу вона харчувалася переважно кукурудзою, мало вживала м’яса. Дефіцит якої речовини у кукурудзі спричинив розвиток хвороби? A person fell ill with pellagra. During the survey, it became known that for a long time she ate mostly corn, she ate little meat. The deficiency of which substance in corn caused the development of the disease ?

Тирозин Tyrosine

Триптофан Tryptophan

Аланін Alanine

Гістидин Histidine

Пролін Proline

31 / 200
При забарвленні бакпрепаратів, виготовлених з харкотиння, методом Ціля-Нільсена виявлено наявність яскраво-червоних паличок, які розміщувалися поодиноко або групами, не чутливі до дії кислот. На живильних середовищах перші ознаки росту з’являються на 10-і5-ту добу. До якої родини відносяться виявлені бактерії? During the staining of bac preparations made from sputum, the presence of bright red rods was found, which were placed singly or in groups, not sensitive to the action of acids, using the Ziel-Nielsen method. media, the first signs of growth appear on the 10th and 5th day. To which family do the detected bacteria belong?

Micobacterium tuberculosis Mycobacterium tuberculosis

Histoplasma dubrosii Histoplasma dubrosii

Yersinia pseudotuberculosis Yersinia pseudotuberculosis

Coxiella burnettii Coxiella burnettii

Klebsiella rhinoscleromatis Klebsiella rhinoscleromatis

32 / 200
Пацієнтці з високим ступенем ожиріння у якості харчової добавки було рекомендовано карнітин для поліпшення 'спалювання'жиру. Яку безпосередню участь бере карнітин у процесі окиснення жирів? A patient with a high degree of obesity was recommended carnitine as a dietary supplement to improve fat 'burning'. What is the direct role of carnitine in the process of fat oxidation?

Бере участь в одній з реакцій бета-окиснення ВЖК Participates in one of the reactions of beta-oxidation of VHD

Транспорт ВЖК з жирових депо до тканин Transport of UVC from fat depots to tissues

Активація внутрішньоклітинного ліполізу Activation of intracellular lipolysis

Транспорт ВЖК з цитозоля до мітохондрій Transport of UVK from cytosol to mitochondria

Активація ВЖК Activation of LCD

33 / 200
У дитини 2-х років з катаральними явищами та висипом на шкірі лікар запідозрив скарлатину. Внутрішньошкірно дитині ввели невелику кількість сироватки до еритрогенного токсину стрептокока, на місці ін’єкції висип зник. Що означають результати реакції? The doctor suspected scarlet fever in a 2-year-old child with catarrhal symptoms and a rash on the skin. The child was injected intradermally with a small amount of serum to erythrogenic streptococcus toxin, at the injection site The rash is gone. What do the reaction results mean?

Клінічний діагноз підтвердився Clinical diagnosis confirmed

Всю дозу сироватки можна вводити внутрішньовенно The entire dose of serum can be administered intravenously

Імунна система дитини дуже ослаблена The child's immune system is very weak

У дитини підвищена чутливість до еритрогенного токсину The child has increased sensitivity to erythrogenic toxin

Захворювання викликав не гемолітичний стрептокок The disease was not caused by hemolytic streptococcus

34 / 200
У хворого поперечний розрив спинного мозку нижче VI грудного сегменту. Як внаслідок цього зміниться дихання? The patient has a transverse rupture of the spinal cord below the VI thoracic segment. How will breathing change as a result?

Стане більш глибоким Will get deeper

Стане більш рідким Will become rarer

Суттєво не зміниться Will not change significantly

Зупиниться Stop

Стане більш частим Will become more frequent

35 / 200
У хворої на бронхіальну астму вірусне інфікування спровокувало астматичний статус зі смертельним наслідком. При гістологічному дослідженні легень виявлено: спазм і набрякання бронхіол, в їх стінках виражена інфільтрація лімфоцитами, еозинофілами та іншими лейкоцитами, а також дегрануляція лаброцитів. Який механізм гіперчутливості лежить в основі зазначених змін? In a patient with bronchial asthma, a viral infection provoked an asthmatic status with a fatal outcome. Histological examination of the lungs revealed: spasm and swelling of the bronchioles, marked infiltration of lymphocytes and eosinophils in their walls and other leukocytes, as well as degranulation of labrocytes. What mechanism of hypersensitivity underlies these changes?

Імунокомплексний Immunocomplex

Реагінова реакція Reagin reaction

Аутоімунний Autoimmune

Імунозумовлений клітинний цитоліз Immune-induced cellular cytolysis

Запальний Incendiary

36 / 200
Мати виявила у 5-річної доньки на періанальних складках білих 'черв’ячків ', які викликали у дитини свербіж і неспокій, і доставила їх до лабораторії. Під час огляду лікар побачив білих гельмінтів 0,5-1 см довжиною, ниткоподібної форми з загостреними кінцями, у деяких кінці були закручені. Який найбільш вірогідний діагноз? The mother found white 'worms' on the perianal folds of her 5-year-old daughter, which caused the child itching and restlessness, and took them to the laboratory. During during the examination, the doctor saw white helminths 0.5-1 cm long, thread-like in shape with pointed ends, some of the ends were twisted. What is the most likely diagnosis?

Дифілоботріоз Diphyllobotriosis

Теніоз Taeniosis

Опісторхоз Opistorchosis

Аскаридоз Ascariasis

Ентеробіоз Enterobiosis

37 / 200
У хворого 27-ми років виявлено патологічні зміни печінки і головного мозку. У плазмі крові виявлено різке зниження, а в сечі - підвищення вмісту міді. Встановлено діагноз - хвороба Вільсона. Активність якого ферменту в сироватці крові необхідно дослідити для підтвердження діагнозу? A 27-year-old patient was diagnosed with pathological changes in the liver and brain. A sharp decrease was detected in the blood plasma, and an increase in the copper content in the urine. The diagnosis was made - the disease Wilson. The activity of which enzyme in blood serum should be investigated to confirm the diagnosis?

Лейцинамінопептидаза Leucinaminopeptidase

Церулоплазмін Ceruloplasmin

Ксантиноксидаза Xanthine oxidase

Карбоангідраза Carbonic anhydrase

Алкогольдегідрогеназа Alcohol dehydrogenase

38 / 200
Пацієнт звернувся до лікаря зі скаргами на задишку, що виникала після фізичного навантаження. Клінічне обстеження виявило анемію та наявність парапротеїну в зоні гамма-глобулінів. Який показник у сечі необхідно визначити для підтвердження діагнозу мієломи? The patient turned to the doctor with complaints of shortness of breath that occurred after physical exertion. Clinical examination revealed anemia and the presence of paraprotein in the gamma globulin zone. What indicator in the urine is necessary determine to confirm the diagnosis of myeloma?

Антитрипсин Antitrypsin

Білірубін Bilirubin

Білок Бенс-Джонса Bence-Jones protein

Церулоплазмін Ceruloplasmin

Гемоглобін Hemoglobin

39 / 200
У чоловіка 45-ти років після зна- чного психоемоційного навантаження раптово з’явився стискаючий біль в ділянці серця з іррадіацією в ліву руку, шию, під ліву лопатку. Обличчя стало блідим, вкрилося холодним потом. Нітрогліцерин усунув напад болю. Який процес розвинувся у хворого? A 45-year-old man, after significant psycho-emotional stress, suddenly developed a squeezing pain in the area of the heart with radiation to the left arm, neck, under the left shoulder blade. The face became pale, covered with cold sweat. Nitroglycerin eliminated the pain attack. What process developed in the patient?

Інфаркт міокарда Myocardial infarction

Перфорація виразки шлунка Perforation of gastric ulcer

Інсульт Stroke

Стенокардія Angina

Психогенний шок Psychogenic shock

40 / 200
Жінка 62-х років скаржиться на частий біль у ділянці грудної клітки та хребта, переломи ребер. Лікар припустив мієломну хворобу (плазмоцитому). Який з перерахованих нижче лабораторних показників буде мати найбільше діагностичне значення? A 62-year-old woman complains of frequent chest and spine pain, rib fractures. The doctor suspected myeloma (plasmocytoma). Which of the following laboratory indicators will have the greatest diagnostic value?

Гіпоглобулінемія Hypoglobulinemia

Гіпопротеїнемія Hypoproteinemia

Гіперальбумінемія Hyperalbuminemia

Протеїнурія Proteinuria

Парапротеїнемія Paraproteinemia

41 / 200
Дитині 8-ми років, що надійшла до інфекційного відділення з високою температурою (до 38oC), дрібноточковим яскраво-червоним висипом, було встановлено діагноз скарлатини. Об’єктивно: слизова оболонка зіву яскраво гіперемована, набрякла, мигдалики різко збільшені, з тьмяними вогнищами жовтувато-сірого кольору і ділянками чорного кольору. Яке запалення лежить в основі змін у зіві? An 8-year-old child who came to the infectious disease department with a high temperature (up to 38oC), small-point bright red rash, was diagnosed with scarlet fever. Objectively : the mucous membrane of the pharynx is highly hyperemic, swollen, the tonsils are sharply enlarged, with dull yellowish-gray foci and areas of black color. What kind of inflammation is the basis of the changes in the pharynx?

Геморагічне Hemorrhagic

Катаральне Catarrhal

Фібринозне Fibrinous

Гнійно-некротичне Purulent-necrotic

Серозне Serious

42 / 200
У новонародженої дитини на пелюшках виявлені темні плями, що свідчать про утворення гомогентизинової кислоти. З порушенням обміну якої речовини це пов’язане? Dark spots were found on the diapers of a newborn child, indicating the formation of homogentisic acid. What substance is this related to a metabolic disorder?

Галактоза Galactose

Метіонін Methionine

Тирозин Tyrosine

Холестерин Cholesterol

Триптофан Tryptophan

43 / 200
У гістологічному препараті визначаються рецепторна зона сенсоепітеліального органа чуттів. Клітини даної зони лежать на базальній мембрані і включають наступні види: зовнішні та внутрішні сенсорні, зовнішні та внутрішні фалангові, стовбурові, зовнішні межові і зовнішні підтримуючі. Вкажіть, якому органу чуттів належить зазначена рецепторна зона: In the histological preparation, the receptor zone of the sensorepithelial sense organ is determined. The cells of this zone lie on the basement membrane and include the following types: external and internal sensory, external and internal phalangeal, trunk , external boundary and external supporting. Indicate to which sense organ the indicated receptor zone belongs:

Рівноваги Balances

Зору Sight

Слуху Hearing

Нюху Sniff

Смаку Taste

44 / 200
Хворий надійшов до відділення з нападом задухи, зумовленої спазмом гладенької мускулатури дихальних шляхів. Назвіть відділи повітроносних шляхів, зміна стану яких, в основному, спричинила даний напад: The patient came to the department with an attack of suffocation caused by a spasm of the smooth muscles of the respiratory tract. Name the departments of the airways, the change in condition of which mainly caused this attack:

Респіраторний відділ Respiratory Department

Бронхи великого калібру Large caliber bronchi

Кінцеві бронхіоли Terminal bronchioles

Бронхи середнього калібру Bronchi of medium caliber

Бронхи малого калібру Small caliber bronchi

45 / 200
Під час обіду дитина поперхнулася і аспірувала їжу. Почався сильний кашель, шкіра і слизові ціанотичні, пульс частий, дихання рідке, видих подовжений. Яке порушення зовнішнього дихання розвинулося у дитини? During lunch, the child sneezed and aspirated food. A strong cough began, the skin and mucous membranes were cyanotic, the pulse was frequent, breathing was thin, exhalation was prolonged. What disorder of external breathing developed in child?

Експіраторна задишка при асфіксії Expiratory shortness of breath during asphyxia

Дихання Біота Biot's Breath

Стенотичне дихання Stenotic breathing

Альтернуюче дихання Alternating breathing

Інспіраторна задишка при асфіксії Inspiratory shortness of breath during asphyxia

46 / 200
Хворий 50-ти років звернувся до клініки зі скаргами на загальну слабкість, втрату апетиту, аритмію серця. Спостерігається гіпотонія м’язів, мляві паралічі, послаблення перистальтики кишечнику. Причиною такого стану може бути: A 50-year-old patient came to the clinic with complaints of general weakness, loss of appetite, heart arrhythmia. Hypotonia of muscles, lethargic paralysis, weakening of intestinal peristalsis are observed. The reason for this condition can be:

Гіперкаліємія Hyperkalemia

Гіпокаліємія Hypokalemia

Гіпофосфатемія Hypophosphatemia

Гіпонатріємія Hyponatremia

Гіпопротеїнемія Hypoproteinemia

47 / 200
Під час гри у волейбол спортсмен після стрибка приземлився на зовнішній край стопи. Виник гострий біль у гомілковостопному суглобі, активні рухи в ньому обмежені, пасивні - в повному обсязі, але болісні. Потім розвинулася припухлість у ділянці зовнішньої щиколотки, шкіра почервоніла, стала теплішою на дотик. Який вид розладу периферичного кровообігу розвинувся в даному випадку? While playing volleyball, the athlete landed on the outer edge of the foot after a jump. There was a sharp pain in the ankle joint, active movements in it are limited, passive - in full, but painful. Then swelling developed in the area of the outer ankle, the skin reddened, became warmer to the touch. What type of peripheral blood circulation disorder developed in this case?

Артеріальна гіперемія Arterial hyperemia

Венозна гіперемія Venous hyperemia

Емболія Embolism

Тромбоз Thrombosis

Стаз Status

48 / 200
У хворого на міастенію після призначення прозерину з’явилися нудота, діарея, посмикування м’язів язика і скелетних м’язів. Чим можна усунути інтоксикацію? A patient with myasthenia gravis developed nausea, diarrhea, twitching of the tongue and skeletal muscles after prescribing Proserin. What can be done to eliminate intoxication?

Мезатон Mesaton

Фізостигмін Physostigmine

Пірідостигміну бромід Pyridostigmine bromide

Ізадрин Izadrin

Атропіну сульфат Atropine sulfate

49 / 200
Чоловік помер від гострого інфекційного захворювання, яке супроводжувалось гарячкою, жовтяницею, геморагічною висипкою на шкірі та слизових оболонках, а також гострою нирковою недостатністю. При гістологічному дослідженні тканини нирки (забарвлення за Романовським-Гімзою) виявлені звивисті бактерії, які мають вигляд букв С та S. Які бактерії були виявлені? The man died of an acute infectious disease, which was accompanied by fever, jaundice, a hemorrhagic rash on the skin and mucous membranes, as well as acute kidney failure. During histological examination of kidney tissue ( Romanovsky-Giemsa staining) winding bacteria were found that look like the letters C and S. What bacteria were found?

Боррелії Borrelia

Кампілобактерії Campylobacteria

Лептоспіри Leptospira

Спіролли Spirals

Трепонеми Treponemes

50 / 200
У підлітка 12-ти років, який хворіє на бронхіальну астму, виник тяжкий напад астми: виражена експіраторна задишка, блідість шкірних покривів. Який вид порушення альвеолярної вентиляції має місце? A 12-year-old teenager suffering from bronchial asthma had a severe asthma attack: severe expiratory shortness of breath, pallor of the skin. What type of alveolar ventilation disorder occurs ?

Торако-діафрагмальний Thoraco-diaphragmatic

Центральний Central

Рестриктивний Restrictive

Нервово-м’язовий Neuromuscular

Обструктивний Obstructive

51 / 200
Чоловіку 46-ти років, що хворіє на дифузний токсичний зоб, була проведена операція резекції щитоподібної залози. Після операції відмічаються відсутність апетиту, диспепсія, підвищена нервово-м’язова збудливість. Маса тіла не збільшилася. Температура тіла у нормі. Чим, із нижче переліченого, обумовлений стан хворого? A 46-year-old man suffering from a diffuse toxic goiter underwent thyroid resection surgery. After the operation, there was a lack of appetite, dyspepsia, increased nervousness' ulcer excitability. Body weight has not increased. Body temperature is normal. What, from the following, is the cause of the patient's condition?

Підвищенням продукції тиреоліберину Thyroliberin production increase

Підвищенням продукції кальцитоніну Increased production of calcitonin

Зниженням продукції паратгормону Decreased production of parathyroid hormone

Зниженням продукції тироксину Decreased production of thyroxine

Підвищенням продукції тироксину Thyroxin production increase

52 / 200
Хворому, який переніс інфаркт міокарда, призначена ацетилсаліцилова кислота по 75 мг щоденно. З якою метою призначено препарат? A patient who suffered a myocardial infarction was prescribed acetylsalicylic acid 75 mg daily. What is the purpose of the drug?

Зменшення агрегації тромбоцитів Decreased platelet aggregation

Зменшення болю Pain Reduction

Зниження температури Decreasing temperature

Розширення коронарних судин Dilation of coronary vessels

Зменшення запалення Decreasing inflammation

53 / 200
При медичному огляді у військкоматі був виявлений хлопчик 15-ти років, високого зросту, з євнухоїдними пропорціями тіла, гінекомастією, з ростом волосся на лобку за жіночим типом. Відмічається відкладання жиру на стегнах, відсутність росту волосся на обличчі, високий голос; коефіцієнт інтелекту знижений. Виберіть каріотип, що відповідає даному захворюванню: During a medical examination, a 15-year-old boy, tall, with eunuch-like body proportions, gynecomastia, and female-type pubic hair growth was found at the Military Commissariat. fat deposits on the thighs, lack of facial hair, high-pitched voice, low IQ. Select the karyotype that corresponds to this condition:

47, XXY 47, XXY

46, XX 46, XX

45, XO 45, XO

47, XXX 47, XXX

46, XY 46, XY

54 / 200
З метою попередження відторгнення трансплантата після пересадки органів обов’язковим є проведення курсу гормонотерапії з метою імуносупресії. Які гормони застосовують з цією метою? In order to prevent graft rejection after organ transplantation, it is mandatory to conduct a course of hormone therapy for the purpose of immunosuppression. What hormones are used for this purpose?

Мінералокортикоїди Mineralocorticoids

Статеві гормони Sex hormones

Катехоламіни Catecholamines

Глюкокортикоїди Glucocorticoids

Тиреоїдні Thyroid

55 / 200
У хворої 53-х років внаслідок тупої травми живота діагностований розрив печінки. У якому анатомічному утворенні збереться кров, що вилилася? A 53-year-old patient was diagnosed with a rupture of the liver as a result of a blunt abdominal injury. In which anatomical formation will the spilled blood collect?

Сальникова сумка Gland bag

Правий брижовий синус Right mesenteric sinus

Лівий брижовий синус Left mesenteric sinus

Прямокишково-маткове заглиблення Recto-uterine penetration

Міхурово-маткове заглиблення Bulco-uterine depression

56 / 200
Хвора скаржиться на набряк ніг, посиніння шкіри, невеличкі виразки збоку латерального виростку. При обстеженні відмічено: припухлість, збільшення розмірів вен, утворення вузлів. З боку якої вени відмічається патологія? The patient complains of swelling of the legs, blue skin, small ulcers on the side of the lateral condyle. During the examination, swelling, an increase in the size of the veins, the formation of nodes are noted. On which side of the vein is the pathology?

V profunda femoris V profunda femoris

V. saphena parva V. saphena parva

V iliaca externa V iliaca externa

V saphena magna V saphena magna

V femoralis V femoralis

57 / 200
У дитини 1,5 років спостерігається відставання в розумовому і фізичному розвитку, посвітління шкіри і волосся, зниження вмісту в крові катехоламінів. При додаванні до свіжої сечі декількох крапель 5 % розчину трихлороцтового заліза з’являється оливковозелене забарвлення. Для якої патології обміну амінокислот характерні дані зміни? In a 1.5-year-old child, there is a delay in mental and physical development, lightening of the skin and hair, a decrease in the content of catecholamines in the blood. When adding a few drops of 5 % solution of iron trichloroacetate, an olive-green color appears. For which pathology of amino acid metabolism are these changes characteristic?

Ксантонурія Xanthonuria

Фенілкетонурія Phenylketonuria

Тирозиноз Tyrosinosis

Альбінізм Albinism

Алкаптонурія Alkaptonuria

58 / 200
При розтині тіла померлого чоловіка 73-х років, який довго хворів на ішемічну хворобу серця з серцевою недостатністю, знайдено: 'мускатна'печінка, бура індурація легень, ціанотична індурація нирок та селезінки. Вкажіть, який з видів порушення кровообігу в хворого призвів до таких наслідків? At the autopsy of the dead man of 73 years old, who had been suffering from coronary heart disease with heart failure for a long time, the following were found: 'nutmeg' liver, brown induration of the lungs, cyanotic induration of the kidneys and spleen. Specify which of the types of circulatory disorders in the patient led to such consequences?

Хронічне недокрів’я Chronic anemia

Хронічне загальне венозне повнокров’я Chronic general venous congestion

Гостре недокрів’я Acute anemia

Артеріальна гіперемія Arterial hyperemia

Гостре загальне венозне повнокров’я Acute general venous congestion

59 / 200
Хворий з хронічним гіперацидним гастритом для усунення печії застосовує антацидний засіб, після прийому якого відзначає поліпшення, однак разом з тим виникає відчуття розпирання у шлунку. Який з наведених препаратів міг викликати вказаний побічний ефект? A patient with chronic hyperacid gastritis uses an antacid to relieve heartburn, after taking which he notes an improvement, but at the same time there is a feeling of distension in the stomach. Which of the following drugs could cause the specified side effect?

Алюмінію гідроокис Aluminum hydroxide

Магнію окис Magnesium oxide

Пепсин Pepsin

Магнію трисилікат Magnesium trisilicate

Натрію гідрокарбонат Sodium bicarbonate

60 / 200
При тривалому використанні препарату в хворого можуть мати місце остеопороз, ерозії слизової шлунка, гіпокаліємія, затримка натрію i води в організмі, зменшення вмісту кортикотропіну в крові. Укажіть цей препарат: With long-term use of the drug, the patient may experience osteoporosis, erosions of the gastric mucosa, hypokalemia, retention of sodium and water in the body, and a decrease in the content of corticotropin in the blood. Specify this drug :

Індометацин Indomethacin

Преднізолон Prednisone

Гіпотіазид Hypotiazid

Дигоксин Digoxin

Резерпін Reserpin

61 / 200
Хворому тривалий час вводили високі дози гідрокортизону, внаслідок чого настала атрофія однієї з зон кори наднирників. Яка це зона? The patient was administered high doses of hydrocortisone for a long time, as a result of which atrophy of one of the zones of the adrenal cortex occurred. Which zone is this?

- -

Клубочкова і сітчаста Globular and reticular

Пучкова Puchkova

Сітчаста Grid

Клубочкова Klubochkova

62 / 200
До лікарні швидкої допомоги госпіталізований чоловік 63-х років з явищами колапсу. Для боротьби з гіпотензією лікар вибрав норадреналін. Який механізм дії цього препарату? A 63-year-old man with collapse symptoms was admitted to the emergency hospital. To combat hypotension, the doctor chose norepinephrine. What is the mechanism of action of this drug?

Активація серотонінових рецепторів Activation of serotonin receptors

Активація дофамінових рецепторів Activation of dopamine receptors

Активація α1 -адренорецепторів Activation of α1-adrenoceptors

Блокада M-холінорецепторів Blockade of M-cholinergic receptors

Активація β-адренорецепторів Activation of β-adrenoceptors

63 / 200
Основна маса азоту з організму виводиться у вигляді сечовини. Зниження активності якого ферменту в печінці призводить до гальмування синтезу сечовини і нагромадження амоніаку в крові і тканинах? The main mass of nitrogen is excreted from the body in the form of urea. A decrease in the activity of which enzyme in the liver leads to the inhibition of urea synthesis and the accumulation of ammonia in the blood and tissues?

Аспартатамінотрансфераза Aspartate aminotransferase

Карбамоїлфосфатсинтаза Carbamoyl phosphate synthase

Уреаза Urease

Пепсин Pepsin

Амілаза Amylase

64 / 200
У хворого на сифіліс при лікуванні препаратами вісмуту з’явилися сірі плями на слизовій оболонці ротової порожнини та симптоми нефропатії. Який засіб доцільно використати у хворого для лікування отруєння препаратами вісмуту? Gray spots appeared on the mucous membrane of the oral cavity and symptoms of nephropathy in a syphilis patient treated with bismuth drugs. What remedy should be used in the patient to treat bismuth poisoning ?

Налоксон Naloxone

Унітіол Unithiol

Бемегрид Bemegrid

Налорфін Nalorphine

Метиленовий синій Methylene blue

65 / 200
На ЕКГ пацієнта мають місце такі зміни: зубець P - нормальний, інтервал P — Q - вкорочений, шлуночковий комплекс QRST - розширений, зубець R -двогорбий або двофазний. Яка із форм аритмії має місце у даного пацієнта? The patient's ECG shows the following changes: the P wave is normal, the P — Q interval is shortened, the ventricular QRST complex is widened, the R wave is two-humped or biphasic. Which form of arrhythmia does this patient have?

Синдром Фредеріка (тріпотіння передсердь) Frederick syndrome (atrial flutter)

Миготлива аритмія Atrial fibrillation

Синдром WPW (Вольфа-Паркінсона-Уайта) WPW (Wolf-Parkinson-White) syndrome

Миготіння шлуночків Ventricular fibrillation

Атріовентрикулярна блокада Atrioventricular block

66 / 200
У гістологічному препараті паренхіма органа представлена лімфоїдною тканиною, яка утворює лімфатичні вузлики; останні розташовуються дифузно і містять центральну артерію. Яке анатомічне утворення має таку морфологічну будову? In the histological preparation, the parenchyma of the organ is represented by lymphoid tissue that forms lymph nodes; the latter are located diffusely and contain a central artery. What anatomical formation has such a morphological structure?

Лімфатичний вузол Lymph node

Червоний кістковий мозок Red bone marrow

Мигдалик Amygdalik

Селезінка Spleen

Тимус Thymus

67 / 200
У хворого з частими кровотечами з внутрішніх органів і слизових оболонок виявлені пролін і лізин у складі колагенових волокон. Через відсутність якого вітаміну порушено їх гідрокси-лювання? In a patient with frequent bleeding from internal organs and mucous membranes, proline and lysine were found in the composition of collagen fibers. Due to the lack of which vitamin their hydroxylation was disturbed?

Вітамін E Vitamin E

Тіамін Thiamine

Вітамін C Vitamin C

Вітамін K Vitamin K

Вітамін A Vitamin A

68 / 200
При диспансерному обстеженні хлопчику 7-ми років встановлено діагноз - дальтонізм. Батьки здорові, кольоровий зір у них у нормі, але у дідуся по материнській лінії така ж аномалія. Який тип успадкування цієї аномалії? During a dispensary examination, a 7-year-old boy was diagnosed with color blindness. His parents are healthy, their color vision is normal, but his maternal grandfather has the same anomaly. What is the type of inheritance of this anomaly?

Рецесивний, зчеплений зі статтю Sex-linked recessive

Домінантний, зчеплений зі статтю Sex-linked dominant

Аутосомно-рецесивний Autosomal recessive

Неповне домінування Incomplete dominance

Аутосомно-домінантний Autosomal dominant

69 / 200
У здорової дорослої людини швидкість проведення збудження через атріовентрикулярний вузол дорівнює 0,02-0,05 м/с. Атріовентрикулярна затримка забезпечує: In a healthy adult, the speed of excitation through the atrioventricular node is 0.02-0.05 m/s. Atrioventricular delay provides:

Достатню силу скорочення передсердь Sufficient force of atrial contraction

Одночасність скорочення обох шлуночків Simultaneous contraction of both ventricles

Одночасність скорочення обох передсердь Simultaneous contraction of both atria

Послідовність скорочення передсердь та шлуночків Atrial and ventricular contraction sequence

Достатню силу скорочення шлуночків Sufficient force of ventricular contraction

70 / 200
На гістологічному препарат нирки представлена ділянка дистального канальцю нефрону, що проходить між приносною та виносною артеріолами. В клітинах, що складають стінку ка-нальцю, наявні ущільнені ядра, відсутня базальна мембрана. Як зветься це структурне утворення? On the histological preparation of the kidney, a section of the distal tubule of the nephron is presented, passing between the supplying and outgoing arterioles. In the cells that make up the wall of the tubule, there are compacted nuclei, there is no basement membrane. What is the name of this structural formation?

Юкстагломерулярні клітини Juxtaglomerular cells

Клітини Гурмагтига Cells of Gurmagtig

Щільна пляма Dense spot

Мезангіальні клітини Mesangial cells

Юкставаскулярні клітини Juxtavascular cells

71 / 200
Внаслідок дії електричного струму на збудливу клітину виникла деполяризація її мембрани. Вхід яких іонів через мембрану до клітини відіграв основну роль в розвитку деполяризації? As a result of the action of an electric current on an excitable cell, its membrane depolarized. What ions entered the cell through the membrane played the main role in the development of depolarization?

Ca2+ Ca2+

K+ K+

Na+ Na+

Cl Cl

HCO3- HCO3-

72 / 200
Продуктами гідролізу та модифікації деяких білків є біологічно активні речовини - гормони. З якого із наведених білків у гіпофізі утворюються ліпотропін, кортикотропін, меланотропін та ендорфіни? The products of hydrolysis and modification of some proteins are biologically active substances - hormones. Lipotropin, corticotropin, melanotropin and endorphins are formed from which of the following proteins in the pituitary gland?

Нейроглобулін Neuroglobulin

Нейроальбумін Neuroalbumin

Тиреоглобулін Thyroglobulin

Проопіомеланокортин (ПОМК) Proopiomelanocortin (POMK)

Нейростромін Neurostromin

73 / 200
У хворих з непрохідністю жовчовивідних шляхів пригнічується зсідання крові, виникають кровотечі, що є наслідком недостатнього засвоєння такого вітаміну: In patients with obstruction of the biliary tract, blood clotting is inhibited, bleeding occurs, which is a consequence of insufficient absorption of this vitamin:

C C

K K

E E

A A

D D

74 / 200
Хвора 48-ми років надійшла до клініки із скаргами на слабкість, дратівливість, порушення сну. Об’єктивно: шкіра та склери жовтого кольору. У крові: підвищення рівня загального білірубіну з переважанням прямого. Кал - ахолічний. Сеча - темного кольору (жовчні пігменти). Яка жовтяниця має місце в хворої? A 48-year-old patient came to the clinic with complaints of weakness, irritability, sleep disturbances. Objectively: the skin and sclera are yellow. In the blood: increased levels of total bilirubin with a predominance of direct. Stool - acholic. Urine - dark color (bile pigments). What kind of jaundice occurs in the patient?

Синдром Кріглера-Найяра Crigler-Nayar syndrome

Механічна Mechanical

Синдром Жільбера Gilbert Syndrome

Гемолітична Hemolytic

Паренхіматозна Parenchymatous

75 / 200
У баклабораторії під час мікроскопії мазків з харкотиння хворого на хронічне легеневе захворювання, забарвлених за Цілем-Нільсеном, виявлені червоні палички. Яка властивість туберкульозної палички виявлена при цьому? During microscopy of sputum smears of a patient with chronic lung disease, stained according to Ziel-Nielsen, red bacilli were found in the laboratory. What property of the tubercle bacillus was detected?

Лугостійкість Alkali resistance

Кислотостійкість Acid resistance

Капсулоутворення Encapsulation

Спороутворення Spore formation

Спиртостійкість Alcohol resistance

76 / 200
Під час мікроскопічного дослідження збільшеного шийного лімфатичного вузла визначається стертя його структури, лімфоїдні фолікули відсутні, усі поля зору представлені клітинами з округлими ядрами і вузьким обідком базофільної цитоплазми. З клінічних даних відомо, що збільшені і інші групи лімфовузлів, а також селезінка та печінка. Про яке захворювання слід думати? During microscopic examination of an enlarged cervical lymph node, its structure is obliterated, lymphoid follicles are absent, all fields of view are represented by cells with rounded nuclei and a narrow rim of basophilic cytoplasm. From clinical from the data, it is known that other groups of lymph nodes are enlarged, as well as the spleen and liver. What disease should we think about?

Лімфосаркома Lymphosarcoma

Лімфоїдний лейкоз Lymphoid leukemia

Мієломна хвороба Myeloma

Лімфогранулематоз Lymphogranulomatosis

Мієлоїдний лейкоз Myeloid leukemia

77 / 200
У юнака 20-ти років, через 2 тижні після перенесеної лакунарної ангіни, з’явилися скарги на загальну слабкість, набряки під очима. Після обстеження хворому встановлено діагноз: гострий гломерулонефрит. Які патологічні зміни у складі сечі найбільш вірогідні? A young man in his 20s, 2 weeks after experiencing lacunar angina, complained of general weakness, swelling under the eyes. After examination, the patient was diagnosed with: acute glomerulonephritis. What pathological changes in the composition of urine are most likely?

Натрійурія Natriuria

Протеїнурія Proteinuria

Циліндрурія Cylindruria

Наявність свіжих еритроцитів Availability of fresh red blood cells

Піурія Pyuria

78 / 200
До травматологічного пункту звернувся чоловік 38-ми років з травмою правої кисті. При огляді встановлено: різана рана в ділянці підвищення великого пальця правої кисті; дистальна фаланга I пальця не згинається. Який м’яз пошкоджено? A 38-year-old man came to the trauma center with an injury to the right hand. During the examination, it was found: a cut wound in the area of the thumb of the right hand; the distal phalanx of the first finger is not bends. What muscle is damaged?

Довгий м’яз-згинач великого пальця Flexor pollicis longus

Привідний м’яз великого пальця Adductor of the thumb

Протиставний м’яз великого пальця Opposite thumb muscle

Короткий м’яз-згинач великого пальця Flexor pollicis brevis

Короткий відвідний м’яз великого пальця Abductor pollicis brevis

79 / 200
У людини збільшена вентиляція легень внаслідок фізичного навантаження. Який з наведених показників зовнішнього дихання у неї значно більший, ніж у стані спокою? A person has increased ventilation of the lungs as a result of physical exertion. Which of the following indicators of external breathing is significantly greater than at rest?

Резервний об’єм видиху Reserve expiratory volume

Загальна ємність легень Total lung capacity

Дихальний об’єм Respiratory volume

Життєва ємність легень Vital lung capacity

Резервний об’єм вдиху Inspiratory reserve volume

80 / 200
У людини внаслідок тривалого голодування швидкість клубочкової фільтрації зросла на 20%. Найбільш вірогідною причиною змін фільтрації в зазначених умовах є: Due to long-term starvation, the rate of glomerular filtration has increased by 20% in a person. The most likely reason for changes in filtration under the specified conditions is:

Збільшення проникності ниркового фільтру Increase in renal filter permeability

Збільшення ниркового кровотоку Increase in renal blood flow

Зменшення онкотичного тиску плазми крові Reduction of oncotic pressure of blood plasma

Збільшення системного артеріального тиску Increase in systemic blood pressure

Збільшення коефіцієнта фільтрації Increase the filter factor

81 / 200
У пацієнта після переливання 200 мл крові підвищилася температура тіла до 37,9o C. Яка з наведених речовин найбільш вірогідно призвела до підвищення температури? The patient's body temperature rose to 37.9o C after a transfusion of 200 ml of blood. Which of the following substances most likely caused the temperature to rise?

Інтерлейкін-4 Interleukin-4

Фактор некрозу пухлин Tumor necrosis factor

Інтерлейкін-2 Interleukin-2

Інтерлейкін-1 Interleukin-1

Інтерлейкін-3 Interleukin-3

82 / 200
У людини, яка обертається на каруселі, збільшилися частота серцевих скорочень, потовиділення, з’явилася нудота. З подразненням яких рецепторів, перш за все, це пов’язано? A person spinning on a carousel has an increased heart rate, sweating, and nausea. This is primarily due to irritation of which receptors ?

Зорові Visual

Вестибулярні отолітові Vestibular otoliths

Пропріоцептори Proprioceptors

Слухові Aural

Вестибулярні ампулярні Vestibular ampullary

83 / 200
Робітник тваринницької ферми гостро захворів і при наростаючих явищах інтоксикації помер. При розтині встановлено: селезінка збільшена, в’яла, на розрізі темно-вишневого кольору, зішкріб пульпи рясний. М’які мозкові оболонки на склепінні та основі мозку набряклі, просякнуті кров’ю, мають темно-червоний колір ('шапочка кардинала'). Мікроскопічно: серозно-геморагічне запалення оболонок і тканин головного мозку з руйнуванням стінок дрібних судин. Який найбільш вірогідний діагноз? A worker on a livestock farm became acutely ill and died due to increasing symptoms of intoxication. At autopsy, it was found that the spleen was enlarged, flaccid, dark cherry-colored on the cross-section, the scraping of the pulp was abundant . Soft meninges on the vault and base of the brain are swollen, soaked with blood, have a dark red color ('cardinal's cap'). Microscopically: serous-hemorrhagic inflammation of the membranes and tissues of the brain with destruction of the walls of small vessels. What is the most likely diagnosis?

Туляремія Tularemia

Чума Plague

Бруцельоз Brucellosis

Холера Cholera

Сибірка Anthrax

84 / 200
Жінка 49-ти років звернулася до лікаря зі скаргами на підвищену втомлюваність та появу задишки під час фізичного навантаження. На ЕКГ: ЧСС-50/хв, інтервал PQ- подовжений, комплекс QRS - не змінений, кількість зубців P перевищує кількість комплексів QRS. Який вид аритмії у пацієнтки? A 49-year-old woman consulted a doctor with complaints of increased fatigue and shortness of breath during physical exertion. On the ECG: HR-50/min, interval PQ- prolonged, the QRS complex is unchanged, the number of P waves exceeds the number of QRS complexes. What type of arrhythmia does the patient have?

Миготлива аритмія Atrial fibrillation

Синусова брадикардія Sinus bradycardia

Екстрасистолія Extrasystole

Атріовентрикулярна блокада Atrioventricular block

Синоатріальна блокада Sinoatrial block

85 / 200
Хворий на ішемічну хворобу серця, з метою усунення нападів стенокардії, впродовж дня багаторазово приймав препарат, який в подальшому з причини передозування привів до отруєння. Об’єктивно: ціаноз шкіри та слизових оболонок, різке зниження артеріального тиску, тахікардія, пригнічення дихання. У крові підвищений вміст метгемоглобіну. Препарат якої групи приймав хворий? A patient with coronary heart disease, in order to eliminate angina attacks, took a drug multiple times during the day, which later led to poisoning due to an overdose. Objectively: cyanosis of the skin and mucous membranes, a sharp drop in blood pressure, tachycardia, respiratory depression. The content of methemoglobin in the blood is increased. What group of drugs did the patient take?

Блокатори кальцієвих каналів Calcium channel blockers

Міотропні спазмолітики Myotropic antispasmodics

Препарати аденозинового ряду Drugs of the adenosine series

α-адреноблокатори α-blockers

Органічні нітрати Organic nitrates

86 / 200
У хворого з верхнім типом ожиріння тривало відзначалися артеріальна гіпертонія, гіперглікемія, глюкозурія. Смерть настала від крововиливу у головний мозок. Під час патоморфологічного дослідження виявлені базофільна аденома гіпофізу, гіперплазія кори наднирників. Який найбільш вірогідний діагноз? A patient with the upper type of obesity had prolonged arterial hypertension, hyperglycemia, and glucosuria. Death occurred from a brain hemorrhage. During the pathomorphological examination, basophilic adenoma of the pituitary gland, hyperplasia were found adrenal cortex. What is the most likely diagnosis?

Акромегалія Acromegaly

Гіпофізарний нанізм Pituitary dwarfism

Адипозогенітальна дистрофія Adiposogenital dystrophy

Хвороба Іценка-Кушінга Itsenko-Cushing's disease

Цукровий діабет Diabetes

87 / 200
Чоловік середнього віку виїхав до іншої країни на обіцяну йому роботу, але працевлаштуватися тривалий час йому не вдавалося. Які з ендокринних залоз були виснажені у цієї людини найбільше? A middle-aged man left for another country for a job he was promised, but he could not find a job for a long time. Which of the endocrine glands was depleted in this person the most?

Наднирники Adrenal glands

Щитоподібна Thyroid

Підгрудинна Substernal

Прищитоподібні Pystiliformes

Сім’яники Seeds

88 / 200
У дівчинки діагностований адреногенітальний синдром (псевдогермафродитизм). Надмірна секреція яких гормонів наднирників обумовила дану патологію? The girl was diagnosed with adrenogenital syndrome (pseudohermaphroditism). Excessive secretion of which adrenal hormones caused this pathology?

Катехоламіни Catecholamines

Андрогени Androgens

Глюкокортикоїди Glucocorticoids

Мінералокортикоїди Mineralocorticoids

Естрогени Estrogens

89 / 200
У жінки народилась мертва дитина з багатьма вадами розвитку. Яке протозойне захворювання могло спричинити внутрішньоутробну загибель? A woman gave birth to a dead child with many developmental defects. What protozoan disease could have caused fetal death?

Амебіаз Amebiasis

Токсоплазмоз Toxoplasmosis

Лейшманіоз Leishmaniasis

Лямбліоз Giardiasis

Малярія Malaria

90 / 200
У хворого на рак спинки язика виникла сильна кровотеча внаслідок ураження пухлиною дорзальної артерії язика. Яку судину повинен перев’язати лікар для зупинки кровотечі? A patient with cancer of the dorsum of the tongue experienced severe bleeding as a result of tumor damage to the dorsal artery of the tongue. Which vessel should the doctor ligate to stop the bleeding?

Дорзальна артерія язика Dorsal artery of the tongue

Лицева артерія Facial artery

Глибока артерія язика Deep artery of tongue

Язикова артерія Lingual artery

Висхідна артерія глотки Ascending pharyngeal artery

91 / 200
У підлітка 12-ти років, який впродовж 3-х місяців різко схуд, вміст глюкози у крові 50 ммоль/л. У нього розвинулася кома. Який головний механізм її розвитку? A 12-year-old teenager who lost weight dramatically over the course of 3 months has a blood glucose level of 50 mmol/l. He developed a coma. What is the main mechanism its development?

Гіпоксичний Hypoxic

Кетонемічний Ketonemic

Гіперосмолярний Hyperosmolar

Лактацидемічний Lactacidemic

Гіпоглікемічний Hypoglycemic

92 / 200
У хворого на мікросфероцитарну гемолітичну анемію (хворобу Мінковського-Шоффара), внаслідок підвищення проникливості мембрани еритроцитів, у клітину надходять іони натрію та вода. Еритроцити набувають форму сфероцитів і легко руйнуються. Який провідний механізм пошкодження еритроцитів має місце в даному випадку? In a patient with microspherocytic hemolytic anemia (Minkowski-Shofar disease), sodium ions and water enter the cell due to increased permeability of the erythrocyte membrane. The erythrocytes take the form of spherocytes and easily are destroyed. What is the leading mechanism of erythrocyte damage in this case?

Кальцієвий Calcium

Протеїновий Protein

Ацидотичний Acidotic

Електролітно-осмотичний Electrolyte-osmotic

Нуклеїновий Nucleic

93 / 200
На гістологічному препараті представлено кровоносну судину. Внутрішня оболонка складається з ендотелію, підендотелію та внутрішньої еластичної мембрани. Середня оболонка збагачена гладенькими міоцитами. Вкажіть, для якої судини характерні дані морфологічні ознаки: A blood vessel is shown on the histological preparation. The inner membrane consists of endothelium, subendothelium and internal elastic membrane. The middle membrane is enriched with smooth myocytes. Indicate for which vessel the morphological data are characteristic signs:

Артерія еластичного типу Artery of elastic type

Вена безм’язового типу Vein of muscleless type

Артерія м’язового типу Muscular artery

Вена м’язового типу Muscular vein

Капіляр Capillary

94 / 200
Охолодження тіла людини у воді виникає значно швидше, ніж на повітрі. Який шлях тепловіддачі у воді значно ефективніший? Cooling of the human body occurs much faster in water than in air. Which way of heat transfer in water is much more efficient?

Теплопроведення Heat conduction

Конвекція Convection

- -

Випаровування поту Evaporation of sweat

Тепловипромінювання Heat radiation

95 / 200
До лікарні після автокатастрофи надійшов юнак 18-ти років. У травматологічному відділенні виявлені численні травми м’яких тканин обличчя в ділянці медіального кута ока, які призвели до масивної кровотечі. Який артеріальний анастомоз міг бути пошкоджений у цьому регіоні? An 18-year-old man was admitted to the hospital after a car accident. In the trauma department, multiple injuries of the soft tissues of the face in the area of the medial corner of the eye were found, which led to massive bleeding . What arterial anastomosis could be damaged in this region?

a. carotis externa et a. carotis interna a. carotis externa et a. carotis interna

a. carotis externa et a. subclavia a. carotis externa et a. subclavia

a. carotis interna et a. subclavia a. carotis interna et a. subclavia

a. carotis interna et a. ophthalmica a. carotis interna et a. ophthalmica

a. subclavia et a. ophthalmica a. subclavia et a. ophthalmica

96 / 200
Жінці 36-ти років після хірургічного втручання внутрішньовенно ввели концентрований розчин альбуміну. Це спричинило посилений рух води у такому напрямку: After surgery, a 36-year-old woman was intravenously injected with a concentrated solution of albumin. This caused an increased movement of water in the following direction:

Із капілярів до міжклітинної рідини From capillaries to intercellular fluid

З міжклітинної рідини до клітин From intercellular fluid to cells

Змін руху води не відбуватиметься The water flow will not change

З міжклітинної рідини до капілярів From intercellular fluid to capillaries

Із клітин до міжклітинної рідини From cells to intercellular fluid

97 / 200
Під час розтину трупа чоловіка зі злоякісною пухлиною шлунка, що помер від ракової інтоксикації, в задньонижніх відділах легень виявлені щільні сіро-червоного кольору неправильної форми осередки, які виступають над поверхнею розрізу. Мікроскопічно: у просвіті, стінках дрібних бронхів та альвеолах виявляється ексудат, в якому багато нейтрофілів. Про яке захворювання свідчать зміни у легенях померлого? During the autopsy of a man with a malignant tumor of the stomach who died of cancerous intoxication, dense gray-red cells of irregular shape were found in the posterior lower parts of the lungs, which protrude above surface of the cut. Microscopically: exudate with many neutrophils is found in the lumen, walls of small bronchi and alveoli. What disease do the changes in the lungs of the deceased indicate?

Гостра гнійна бронхопневмонія Acute purulent bronchopneumonia

Гостра серозна бронхопневмонія Acute serous bronchopneumonia

Проміжна пневмонія Intermediate pneumonia

Гострий бронхіт Acute bronchitis

Крупозна пневмонія Croup pneumonia

98 / 200
Під час розтину тіла померлої дитини 1,5 років виявлені: геморагічний висип на шкірі, помірна гіперемія та набряк слизової оболонки носоглотки, дрібні крововиливи у слизових оболонках і внутрішніх органах, різкі дистрофічні зміни у печінці, міокарді, гострий некротичний нефроз, масивні крововиливи у наднирниках. Для якого захворювання найбільш характерні виявлені зміни? During the autopsy of the body of a dead 1.5-year-old child, the following were found: a hemorrhagic rash on the skin, moderate hyperemia and swelling of the mucous membrane of the nasopharynx, small hemorrhages in the mucous membranes and internal organs , sharp dystrophic changes in the liver, myocardium, acute necrotic nephrosis, massive hemorrhages in the adrenal glands. For which disease are the most characteristic of the detected changes?

Кір Measles

Скарлатина Scarlatina

Висипний тиф Typhoid

Дифтерія Diphtheria

Менінгококова інфекція Meningococcal infection

99 / 200
Під час огляду дитини 11-ти місяців педіатр виявив викривлення кісток нижніх кінцівок і затримку мінералізації кісток черепа. Нестача якого вітаміну призводить до даної патології? During the examination of an 11-month-old child, the pediatrician found curvature of the bones of the lower limbs and delayed mineralization of the bones of the skull. The lack of which vitamin leads to this pathology?

Холекальциферол Cholecalciferol

Пантотенова кислота Pantothenic acid

Біофлавоноїди Bioflavonoids

Рибофлавін Riboflavin

Тіамін Thiamine

100 / 200
У хворого на хронічний гепатит виявлено значне зниження синтезу і секреції жовчних кислот. Який процес у найбільшій мірі буде порушений у кишечнику цього хворого? In a patient with chronic hepatitis, a significant decrease in the synthesis and secretion of bile acids was detected. What process will be most affected in the intestines of this patient?

Всмоктування гліцерину Glycerin Absorption

Емульгування жирів Emulsification of fats

Травлення вуглеводів Digestion of carbohydrates

Травлення білків Protein digestion

Всмоктування амінокислот Absorption of amino acids

101 / 200
Для вирішення питання ретроспективної діагностики перенесеної бактеріальної дизентерії було призначено серологічне дослідження сироватки крові з метою встановлення титру антитіл до шигел. Яку з перелічених реакцій доцільно використати для цього? To resolve the issue of retrospective diagnosis of transferred bacterial dysentery, a serological examination of blood serum was prescribed to determine the titer of antibodies to Shigella. Which of the listed reactions should be used for this?

Зв’язування комплементу Complement binding

Преципітація Precipitation

Бактеріоліз Bacteriolysis

Гемоліз Hemolysis

Пасивна гемаглютинація Passive hemagglutination

102 / 200
Вивчається робота оперону бактерії. Відбулося звільнення гена-оператора від білка репресора. Безпосередньо після цього в клітині почнеться: The work of the bacterial operon is being studied. The operator gene has been released from the repressor protein. Immediately after that, the cell will begin:

Трансляція Broadcast

Процесінг Processing

Транскрипція Transcription

Реплікація Replication

Репресія Repression

103 / 200
При визначенні енерговитрат організму людини встановлено, що дихальний коефіцієнт дорівнює 1,0. Це означає, що у клітинах досліджуваного переважно окислюються: When determining the energy expenditure of the human body, it was established that the respiratory coefficient is equal to 1.0. This means that the cells of the subject are mainly oxidized:

Жири Fats

Вуглеводи та жири Carbohydrates and Fats

Білки Proteins

Вуглеводи Carbohydrates

Білки і вуглеводи Proteins and carbohydrates

104 / 200
У тварини в експерименті перерізали задні корінці спинного мозку. Які зміні відбуватимуться в зоні іннервації? In the experiment, the rear roots of the spinal cord were cut in the animal. What changes will occur in the innervation zone?

Втрата рухових функцій Loss of motor functions

Втрата чутливості і рухових функцій Loss of sensitivity and motor functions

Зниження тонусу м’язів Decreased muscle tone

Втрата чутливості Loss of sensitivity

Підвищення тонусу м’язів Increasing muscle tone

105 / 200
Внаслідок руйнування певних структур стовбуру мозку тварина втратила орієнтувальні рефлекси. Які структури було зруйновано? Due to the destruction of certain structures of the brain stem, the animal lost orientation reflexes. What structures were destroyed?

Чотиригорбкова структура Four hump structure

Вестибулярні ядра Vestibular nuclei

Чорна речовина Black Matter

Медіальні ядра ретикулярної формації Medial nuclei of the reticular formation

Червоні ядра Red kernels

106 / 200
У людини осмотичний тиск плазми крові 350 мосмоль/л (норма - 300 мосмоль/л). Це спричинить, перш за все, посилену секрецію такого гормону: In a person, the osmotic pressure of the blood plasma is 350 mosmol/l (the norm is 300 mosmol/l). This will cause, first of all, increased secretion of such a hormone:

Кортизол Cortisol

Вазопресин Vasopressin

Натрійуретичний Natriuretic

Адренокортикотропін Adrenocorticotropin

Альдостерон Aldosterone

107 / 200
Під час бігу на короткі дистанції у нетренованої людини виникає м’язова гіпоксія. До накопичення якого метаболіту в м’язах це призводить? During short-distance running in an untrained person, muscle hypoxia occurs. What metabolite does this lead to the accumulation of in the muscles?

Оксалоацетат Oxaloacetate

Кетонові тіла Ketone bodies

Глюкозо-6-фосфат Glucose-6-phosphate

Лактат Lactate

Ацетил-КоА Acetyl-CoA

108 / 200
У цитоплазмі міоцитів розчинена велика кількість метаболітів окиснення глюкози. Назвіть один з них, який безпосередньо перетворюється на лактат: A large number of metabolites of glucose oxidation are dissolved in the cytoplasm of myocytes. Name one of them that is directly converted to lactate:

Гліцерофосфат Glycerophosphate

Оксалоацетат Oxaloacetate

Піруват Pyruvate

Фруктозо-6-фосфат Fructose-6-phosphate

Глюкозо-6-фосфат Glucose-6-phosphate

109 / 200
Молодий чоловік звернувся до лікарні зі скаргами на порушення сечовипускання. Під час обстеження зовнішніх статевих органів виявлено, що сечівник розщеплений зверху і сеча витікає через цей отвір. Який вид аномалії розвитку зовнішніх статевих органів спостерігається у цьому випадку? A young man went to the hospital with complaints of urination disorders. During the examination of the external genitalia, it was found that the urethra is split from above and urine is leaking through this opening. What kind of abnormality the development of the external genitalia is observed in this case?

Епіспадія Epispadia

Гіпоспадія Hypospadia

Фімоз Phimosis

Парафімоз Paraphimosis

Гермафродитизм Hermaphroditism

110 / 200
Хворий звернувся до лікаря-уролога зі скаргами на біль під час сечовипускання. У сечі, що отримана на аналіз у денний час, були виявлені яйця з характерним шипом. З анамнезу відомо, що хворий недавно повернувся з Австралії. Який найбільш вірогідний діагноз? The patient turned to a urologist with complaints of pain during urination. Eggs with a characteristic spike were found in the urine obtained for analysis during the day. it is known from the anamnesis that the patient recently returned from Australia. What is the most likely diagnosis?

Опісторхоз Opistorchosis

Дикроцеліоз Dicroceliosis

Шистосомоз урогенітальний Urogenital schistosomiasis

Шистосомоз кишковий Intestinal schistosomiasis

Шистосомоз японський Japanese schistosomiasis

111 / 200
У хворого на дизентерію при колоноскопії виявлено, що слизова оболонка товстої кишки гіперемована, набрякла, її поверхня вкрита сіро-зеленими плівками. Назвіть морфологічну форму дизентерійного коліту: In a patient with dysentery, colonoscopy revealed that the mucous membrane of the large intestine is hyperemic, swollen, and its surface is covered with gray-green films. Name the morphological form of dysenteric colitis:

Фібринозний Fibrinous

Некротичний Necrotic

Виразковий Ulcerous

Катаральний Catarrhal

Гнійний Suppurative

112 / 200
У хворого через добу після апендектомії у крові визначається нейтрофільний лейкоцитоз із регенеративним зсувом. Який найбільш вірогідний механізм розвитку лейкоцитозу в даному випадку? One day after appendectomy, neutrophilic leukocytosis with a regenerative shift is detected in the patient's blood. What is the most likely mechanism of development of leukocytosis in this case?

Посилення лейкопоезу та уповільнення міграції лейкоцитів у тканини Increasing leukopoiesis and slowing the migration of leukocytes into tissues

Уповільнення міграції лейкоцитів у тканини Slowing down the migration of leukocytes into tissues

Перерозподіл лейкоцитів у організмі Redistribution of leukocytes in the body

Уповільнення руйнування лейкоцитів Slowing down the destruction of leukocytes

Посилення лейкопоезу Increasing leukopoiesis

113 / 200
Хворий 35-ти років звернувся до лікаря із скаргами на сильний нежить та втрату відчуття запахів протягом тижня. Об’єктивно: в носовій порожнині велика кількість слизу, що вкриває слизову оболонку та блокує рецептори нюху. Де в носовій порожнині розташовані ці рецептори? A 35-year-old patient turned to the doctor complaining of a severe runny nose and loss of sense of smell for a week. Objectively: a large amount of mucus covering the nasal cavity mucous membrane and blocks the olfactory receptors. Where are these receptors located in the nasal cavity?

Середня носова раковина Average turbinate

Загальний носовий хід Common nasal passage

Нижня носова раковина Inferior turbinate

Присінок носа Nose hair

Верхня носова раковина Superior turbinate

114 / 200
У дитини 10-ти років поставлено пробу Манту (з туберкуліном). Через 48 годин на місці введення туберкуліну з’явилася папула розміром до 8 мм у діаметрі. Який тип реакції гіперчутливості розвинувся після введення туберкуліну? A 10-year-old child underwent a Mantoux test (with tuberculin). After 48 hours, a papule up to 8 mm in diameter appeared at the site of tuberculin injection. What type of hypersensitivity reaction developed after administration of tuberculin?

Реакція типу сироваткової хвороби Serum sickness type reaction

Реакція гіперчутливості II типу Type II hypersensitivity reaction

Атопічна реакція Atopic reaction

Реакція типу феномен Артюса Arthus phenomenon type reaction

Реакція гіперчутливості IV типу Type IV hypersensitivity reaction

115 / 200
вітаміну В1 115. порушується окисне декарбоксилювання α-кетоглутарової кислоти. Синтез якого з наведених коферментів порушується при цьому? vitamin B1 115. the oxidative decarboxylation of α-ketoglutaric acid is disturbed. The synthesis of which of the following coenzymes is disturbed at the same time?

Ліпоєва кислота Lipoic acid

Нікотинамідаденіндинуклеотид Nicotinamide adenine dinucleotide

Тіамінпірофосфат Thiamine pyrophosphate

Коензим А Coenzyme A

Флавінаденіндинуклеотид Flavinadenine dinucleotide

116 / 200
З урахуванням клінічної картини хворому призначено піридоксальфосфат. Для корекції яких процесів рекомендований цей препарат? Taking into account the clinical picture, the patient is prescribed pyridoxal phosphate. What processes is this drug recommended for correction?

Окисне декарбоксилювання кетокислот Oxidative decarboxylation of ketoacids

Синтез білку Protein synthesis

Дезамінування пуринових нуклеотидів Deamination of purine nucleotides

Трансамінування і декарбоксилювання амінокислот Transamination and decarboxylation of amino acids

Синтез пуринових та піримідинових основ Synthesis of purine and pyrimidine bases

117 / 200
У хворого з клінічними ознаками імунодефіциту проведено імунологічні дослідження. Виявлено значне зниження кількості клітин, що утворюють розетки з еритроцитами барана. Який висновок слід зробити аналізуючи дані дослідження? In a patient with clinical signs of immunodeficiency, immunological studies were performed. A significant decrease in the number of cells forming rosettes with ram erythrocytes was revealed. What conclusion should be drawn by analyzing the research data?

Зниження рівня системи комплементу Complement system downgrade

Зниження рівня T-лімфоцитів Decrease in the level of T-lymphocytes

Зниження рівня В-лімфоцитів Decrease in the level of B-lymphocytes

Зниження рівня натуральних кілерів (NK-клітин) Decrease in the level of natural killers (NK cells)

Недостатність клітин-ефекторів гуморального імунітету Deficiency of effector cells of humoral immunity

118 / 200
У померлого, що понад 20-ти років працював на вугільній шахті, при розтині тіла знайдені ущільнені легені сіро-чорного кольору зі значними ділянками новоутвореної сполучної тканини та наявністю великої кількості макрофагів з пігментом чорного кольору у цитоплазмі. Який з перелічених діагнозів найбільш вірогідний? In the deceased, who worked in a coal mine for more than 20 years, during the autopsy, compacted gray-black lungs with significant areas of newly formed connective tissue and the presence of a large the number of macrophages with black pigment in the cytoplasm. Which of the listed diagnoses is the most likely?

Антракосилікоз Anthracosilicosis

Сидероз Siderosis

Талькоз Talkoz

Антракоз Anthracosis

Силікоантракоз Silicoanthracosis

119 / 200
На розтині тіла померлого від сепсису, в стегновій кістці нижньої кінцівки виявлено флегмонозне запалення, що охоплює кістковий мозок, гаверсові канали та періост. Під періостом - множинні абсцеси, в навколишніх м’яких тканинах стегна - також флегмонозне запалення. Який патологічний процес має місце? At the autopsy of the body of the deceased from sepsis, phlegmonous inflammation was found in the femur of the lower limb, covering the bone marrow, Haversian canals and periosteum. Under the periosteum - multiple abscesses, in surrounding soft tissues of the thigh - also phlegmonous inflammation. What pathological process is taking place?

Хронічний гематогенний остеомієліт Chronic hematogenous osteomyelitis

- -

Остеопороз Osteoporosis

Остеопетроз Osteopetrosis

Гострий гематогенний остеомієліт Acute hematogenous osteomyelitis

120 / 200
Після попередньої сенсибілізації експериментальній тварині підшкірно ввели дозу антигену. У місці ін’єкції розвинулось фібринозне запалення з альтерацією стінок судин, основної речовини та волокнистих структур сполучної тканини у вигляді мукоїдного та фібриноїдного набухання і некрозу. Яка імунологічна реакція має місцє? After preliminary sensitization, the experimental animal was subcutaneously injected with a dose of antigen. At the injection site, fibrinous inflammation developed with alteration of vessel walls, the main substance, and fibrous structures of connective tissue in the form of mucoid and fibrinoid swelling and necrosis. What immunological reaction is involved?

Нормергічна реакція Normergic reaction

Гіперчутливість негайного типу Immediate Hypersensitivity

Гіперчутливість сповільненого типу Delayed type hypersensitivity

Реакція трансплантаційного імунітету Reaction of transplant immunity

Гранулематоз Granulomatosis

121 / 200
Стоматолог під час огляду порожнини рота на межі середньої і задньої третини спинки язика виявив запалені сосочки. Які сосочки язика запалені? During the examination of the oral cavity, the dentist found inflamed papillae on the border of the middle and back third of the back of the tongue. Which papillae of the tongue are inflamed?

Papillae conicae Papillae conicae

Papillae foliatae Papillae foliatae

Papillae vallatae Papillae vallatae

Papillae fungiformes Papillae fungiformes

Papillae filiformes Papillae filiformes

122 / 200
При розтині трупа чоловіка 50-ти років виявлено наступні зміни: права легеня у всіх відділах помірно щільна, на розрізі тканина безповітряна, дрібнозерниста, сухувата. Вісцеральна плевра з нашаруванням фібрину сіро-коричневого кольору. Який найбільш вірогідний діагноз? The autopsy of a 50-year-old man revealed the following changes: the right lung is moderately dense in all sections, the tissue is airless, fine-grained, dry on cross-section. Visceral pleura with layering gray-brown fibrin. What is the most likely diagnosis?

Туберкульоз Tuberculosis

Інтерстиціальна пневмонія Interstitial pneumonia

Крупозна пневмонія Croup pneumonia

Пневмофіброз Pneumofibrosis

Бронхопневмонія Bronchopneumonia

123 / 200
Під час статевого дозрівання клітини чоловічих статевих залоз починають продукувати чоловічий статевий гормон тестостерон, який обумовлює появу вторинних статевих ознак. Які клітини чоловічих статевих залоз продукують цей гормон? During puberty, the cells of the male gonads begin to produce the male sex hormone testosterone, which determines the appearance of secondary sexual characteristics. What cells of the male gonads produce this hormone?

Клітини Сертолі Sertoli cells

Сперматозоїди Sperm

Сустентоцити Sustentocytes

Підтримуючі клітини Supporting cells

Клітини Лейдіга Leydig cells

124 / 200
При огляді пацієнта виявлене надмірне розростання кісток і м’яких тканин обличчя, збільшені розміри язика, розширені міжзубні проміжки в збільшеній зубній дузі. Які зміни секреції гормонів у нього найбільш вірогідні? During the examination of the patient, excessive growth of the bones and soft tissues of the face, increased tongue size, widened interdental spaces in the enlarged dental arch were found. What changes in hormone secretion do he have the most probable?

Зменшена секреція інсуліну Decreased insulin secretion

Зменшена секреція тироксину Decreased secretion of thyroxine

Зменшена секреція соматотропного гормону Decreased secretion of somatotropic hormone

Збільшена секреція соматотропного гормону Increased secretion of somatotropic hormone

Збільшена секреція інсуліну Increased insulin secretion

125 / 200
На розтині тіла померлого від ниркової недостатності, який протягом останніх 5-ти років хворів на бронхоектатичну хворобу, виявлені збільшені в розмірах нирки щільної консистенції з потовщеним кірковим шаром білого кольору та сальним блиском. Про яке захворювання нирок можна думати? At the autopsy of the body of the deceased from kidney failure, who had been suffering from bronchiectasis for the past 5 years, enlarged kidneys of a dense consistency with a thickened white cortex were found and a greasy shine. What kind of kidney disease can you think of?

- -

Вторинний амілоїдоз Secondary amyloidosis

Хронічний пієлонефрит Chronic pyelonephritis

Гломерулонефрит Glomerulonephritis

Некротичний нефроз Necrotic nephrosis

126 / 200
На розтині тіла жінки 49-ти років, що померла від хронічної ниркової недостатності, виявлено: нирки ущільнені, зменшені, строкаті, з ділянками крововиливів. Мікроскопічно: у ядрах епітелію канальців гематоксилінові тільця, потовщення базальних мембран капілярів клубочків, які мають вигляд дротяних петель, подекуди в капілярах - гіалінові тромби та вогнища фібриноїдного некрозу. Який найбільш вірогідний діагноз? The autopsy of a 49-year-old woman who died of chronic renal failure revealed: the kidneys were compacted, reduced, variegated, with areas of hemorrhage. Microscopically: in the nuclei hematoxylin bodies in the epithelium of the tubules, thickening of the basal membranes of the capillaries of the glomeruli, which look like wire loops, in some places in the capillaries there are hyaline thrombi and foci of fibrinoid necrosis. What is the most likely diagnosis?

Системний червоний вовчак Systemic lupus erythematosus

Ревматизм Rheumatism

Атеросклеротичний нефросклероз Atherosclerotic nephrosclerosis

Амілоїдоз Amyloidosis

Артеріосклеротичний нефросклероз Arteriosclerotic nephrosclerosis

127 / 200
У хворого 37-ми років внаслідок тривалої антибіотикотерапії розвинувся дисбактеріоз кишечнику. Який вид препаратів необхідно використати для нормалізації кишкової мікрофлори? A 37-year-old patient developed intestinal dysbacteriosis as a result of long-term antibiotic therapy. What type of drugs should be used to normalize the intestinal microflora?

Бактеріофаги Bacteriophages

Еубіотики Eubiotics

Сульфаніламіди Sulfonamides

Вітаміни Vitamins

Аутовакцини Autovaccines

128 / 200
До генетичної консультації звернулася сімейна пара, в якій чоловік хворіє на інсулінозалежний цукровий діабет, а жінка здорова. Яка вірогідність появи інсулінозалежного діабету у дитини цього подружжя? A married couple, in which the husband has insulin-dependent diabetes and the woman is healthy, turned to genetic counseling. What is the probability of insulin-dependent diabetes in the child of this couple?

Нижче, ніж в популяції Lower than population

Така сама, як в популяції The same as in the population

Більше, ніж в популяції More than in the population

100% 100%

50% 50%

129 / 200
Хвора похилого віку хворіє на цукровий діабет 2-го типу, який супроводжується ожирінням, атеросклерозом, ішемічною хворобою серця. При цьому виявлена базальна гіперінсулінемія. Запропонуйте хворій адекватне лікування: An elderly patient suffers from type 2 diabetes, which is accompanied by obesity, atherosclerosis, coronary heart disease. At the same time, basal hyperinsulinemia is detected. Offer the patient adequate treatment:

Глібенкламід Glibenclamide

Амлодипін Amlodipine

Ловастатин Lovastatin

Ретаболіл Retabolil

Інсулін Insulin

130 / 200
До лікаря звернулася жінка 32-х років зі скаргами на відсутність лактації після народження дитини. Дефіцитом якого гормону можна пояснити дане порушення? A 32-year-old woman came to the doctor complaining of lack of lactation after giving birth to a child. What hormone deficiency can explain this disorder?

Соматотропін Somatotropin

Пролактин Prolactin

Глюкагон Glucagon

Вазопресин Vasopressin

Тиреокальцитонін Thyrocalcitonin

131 / 200
У хворої під час профілактичного обстеження на медіальній стінці лівої пахвової западини виявлений збільшений лімфовузол метастатичного походження. Вкажіть найбільш вірогідну локалізацію первинної пухлини: During a preventive examination, an enlarged lymph node of metastatic origin was found on the medial wall of the left axilla in the patient. Specify the most likely localization of the primary tumor:

Легеня Lung

Щитоподібна залоза Thyroid

Молочна залоза Mammary gland

Шлунок Stomach

Піднижньощелепна слинна залоза Submandibular salivary gland

132 / 200
У чоловіка, що хворіє на остеохондроз, з’явився різкий біль у м’язах живота (бічних та передніх). При об’єктивному обстеженні лікар констатував підвищену больову чутливість шкіри підчеревної ділянки. Ураження якого нерва могло спричинити цій біль? A man suffering from osteochondrosis developed sharp pain in the abdominal muscles (lateral and front). During an objective examination, the doctor noted increased pain sensitivity of the skin of the subcutaneous area. Damage to which nerve could cause this pain?

Стегновий Femoral

Затульний Sheltered

Сідничний Gluteal

Клубово-підчеревний Iliohyoid

Статево-стегновий Sex-femoral

133 / 200
На рентгенограмі нирок при пієлографії лікар виявив ниркову миску, в яку безпосередньо впадали малі чашечки (великі були відсутні). Яку форму сечовивідних шляхів нирки виявив лікар? On the x-ray of the kidneys during pyelography, the doctor found a renal bowl into which the small calyces directly flowed (large ones were absent). What shape of the urinary tract of the kidney did the doctor find?

Деревоподібна Tree-like

Ампулярна Ampullary

Ембріональна Embryonic

Зріла Mature

Фетальна Fetal

134 / 200
На аутопсії померлого від грипу чоловіка відзначено, що серце дещо збільшене у розмірах, пастозне, на розрізі міокард тьмяний, з крапом. Мікроскопічно: у міокарді на всьому протязі ознаки паренхіматозної жирової і гідропічної дистрофії, строма набрякла, з незначною макрофагально-лімфоцитарною інфільтрацією, судини повнокровні; периваскулярно - петехіальні крововиливи. Який вид міокардиту розвинувся в даному випадку? At the autopsy of a man who died from the flu, it was noted that the heart was somewhat enlarged, pasty, on cross-section the myocardium was dull, with specks. Microscopically: the myocardium showed signs throughout of parenchymal fatty and hydropic dystrophy, the stroma is swollen, with slight macrophage-lymphocyte infiltration, the vessels are full of blood; perivascular - petechial hemorrhages. What type of myocarditis developed in this case?

Серозний вогнищевий Serous focal

Проміжний проліферативний Intermediate proliferative

Гранулематозний Granulomatous

Серозний дифузний Serous diffuse

Гнійний Suppurative

135 / 200
У хворого на хронічну серцеву недостатність, незважаючи на терапію кардіотонічними засобами і тіазидовим діуретиком, зберігаються набряки і виникла загроза асциту. Який препарат слід призначити для підсилення діуретичного ефекту застосованих ліків? In a patient with chronic heart failure, despite therapy with cardiotonic agents and a thiazide diuretic, edema persists and there is a threat of ascites. What drug should be prescribed to enhance the diuretic effect of the drugs used ?

Манітол Mannitol

Спіронолактон Spironolactone

Амілорид Amiloride

Фуросемід Furosemide

Клопамід Clopamide

136 / 200
При гінекологічному огляді жінки 30-ти років на шийці матки виявлені яскраво-червоні блискучі плями, які при дотику легко кровоточать. На біопсії: шматочок шийки матки вкритий циліндричним епітелієм із сосочковими виростами, в товщині тканини розростання залоз. Яка патологія шийки матки виявлена? During a gynecological examination of a 30-year-old woman, bright red shiny spots were found on the cervix, which bleed easily when touched. On biopsy: a piece of the cervix is covered with cylindrical epithelium with papillary growths, in the thickness of the tissue growth of glands. What pathology of the cervix was detected?

Лейкоплакія Leukoplakia

Залозиста гіперплазія Glandular hyperplasia

Справжня ерозія True erosion

Ендоцервіцит Endocervicitis

Псевдоерозія Pseudo-erosion

137 / 200
У мертвонародженої дитини шкіра потовщена, нагадує панцир черепахи, вушні раковини недорозвинені. Гістологічно в шкірі: надмірне зроговіння, атрофiя зернистого шару епідермісу, відсутні запальні зміни. Яке захворювання найбільш вірогідно? In a stillborn child, the skin is thickened, resembling a turtle shell, the auricles are underdeveloped. Histologically, the skin: excessive keratinization, atrophy of the granular layer of the epidermis, no inflammatory changes. What disease is most authentically?

Лейкоплакія Leukoplakia

Дерматоміозит Dermatomyositis

Ксеродермія Xeroderma

Еритроплакія Erythroplakia

Іхтіоз Ichthyosis

138 / 200
У хворого спостерігається погіршення сутінкового зору. Який з вітамінних препаратів слід призначити пацієнту? The patient has a deterioration of twilight vision. Which of the vitamin preparations should be prescribed to the patient?

Кислота нікотинова Nicotinic acid

Кислота аскорбінова Ascorbic acid

Ретинолу ацетат Retinol Acetate

Ціанокобаламін Cyanocobalamin

Піридоксину гідрохлорид Pyridoxine hydrochloride

139 / 200
Хвора звернулась до лікаря із скаргами на біль та обмеження рухів у колінних суглобах. Який з нестероїдних протизапальних засобів краще призначити, враховуючи наявність в анамнезі хронічного гастродуоденіту? The patient turned to the doctor with complaints of pain and limitation of movement in the knee joints. Which of the non-steroidal anti-inflammatory drugs is better to prescribe, taking into account the presence of chronic gastroduodenitis in the anamnesis?

Промедол Promedol

Кислота ацетилсаліцилова Acetylsalicylic acid

Бутадіон Butadione

Целекоксиб Celecoxib

Диклофенак-натрій Diclofenac sodium

140 / 200
Депресії та емоційні розлади є наслідком нестачі у головному мозку норадреналіну, серотоніну та інших біогенних амінів. Збільшення їх вмісту у синапсах можна досягти за рахунок антидепресантів, які гальмують такий фермент: Depression and emotional disorders are the result of a lack of norepinephrine, serotonin and other biogenic amines in the brain. An increase in their content in synapses can be achieved due to antidepressants that inhibit this enzyme :

Оксидаза L-амінокислот L-amino acid oxidase

Моноамінооксидаза Monoamine oxidase

Оксидаза D-амінокислот D-amino acid oxidase

Фенілаланін-4-монооксигеназа Phenylalanine-4-monooxygenase

Диамінооксидаза Diamine oxidase

141 / 200
У хворого з’явилися жовтушність шкіри, склер та слизових оболонок. У плазмі крові підвищений рівень загального білірубіну, в калі - рівень стеркобіліну, в сечі - уробіліну. Який вид жовтяниці у хворого? The patient developed yellowing of the skin, sclera, and mucous membranes. The level of total bilirubin in the blood plasma is elevated, the level of stercobilin in the feces, and the level of urobilin in the urine. What type of jaundice in the patient?

Хвороба Жільбера Gilbert's disease

Холестатична Cholestatic

Обтураційна Obstructive

Паренхіматозна Parenchymatous

Гемолітична Hemolytic

142 / 200
До патогістологічої лабораторії доставлено червоподібний відросток товщиною до 2,0 см. Серозна оболонка його тьмяна, потовщена, вкрита жовто-зеленими плівковими нашаруваннями. Стінка в’яла, сіро-червона. Просвіт відростка розширено, заповнено жовто-зеленими масами. При гістологічному дослідженні виявлено, що стінка інфільтрована нейтрофілами. Визначте захворювання апендикса: An appendix with a thickness of up to 2.0 cm was delivered to the pathology laboratory. Its serous membrane is dull, thickened, covered with yellow-green film layers. The wall is weak, gray -red. The lumen of the appendage is expanded, filled with yellow-green masses. Histological examination revealed that the wall is infiltrated by neutrophils. Determine the disease of the appendix:

Гострий простий апендицит Acute simple appendicitis

Гострий поверхневий апендицит Acute superficial appendicitis

Гострий флегмонозний апендицит Acute phlegmonous appendicitis

Гострий гангренозний апендицит Acute gangrenous appendicitis

Хронічний апендицит Chronic appendicitis

143 / 200
Відпочиваючи на дачі, хлопчик знайшов павука з наступними морфологічними особливостями: довжина - 2 см, кулясте черевце чорного кольору, на спинному боці якого видно червоні плямочки у два ряди, чотири пари членистих кінцівок вкриті дрібними чорними волосками. Визначте дане членистоноге: While resting in the country, the boy found a spider with the following morphological features: length - 2 cm, spherical abdomen of black color, on the back side of which red spots in two rows are visible, four pairs of articulated limbs are covered with small black hairs. Identify this arthropod:

Фаланги Falangs

Тарантул Tarantula

Кліщ Tick

Каракурт Karakurt

Скорпіон Scorpio

144 / 200
У хворого внаслідок травми розвинувся травматичний шок, у перебігу якого мали місце наступні порушення: АТ- 140/90 мм рт.ст., Ps- 120/хв. Хворий метушливий, багатослівний, блідий. Якій стадії шоку відповідає цей стан? As a result of the injury, the patient developed traumatic shock, in the course of which the following disorders occurred: BP - 140/90 mm Hg, Ps - 120/min. The patient is fussy, talkative, pale. What stage of shock does this state correspond to?

Торпідна Torped

Латентний період Latent period

Термінальна Terminal

Кінцева Final

Еректильна Erectile

145 / 200
У хворого, що надійшов до хірургічного відділення з ознаками гострого апендициту, виявлені наступні зміни білої крові: загальна кількість лейкоцитів - 16 • 109 /л. Лейкоцитарна формула: б.- 0, е.- 2%, ю.- 2%, п.- 8%, с.- 59%, л.- 25%, м.- 4%. Як класифікуються зазначені зміни? The following changes in white blood were detected in a patient who was admitted to the surgical department with signs of acute appendicitis: the total number of leukocytes is 16 • 109 /l. Leukocyte formula: b .- 0, e.- 2%, yu.- 2%, n.- 8%, s.- 59%, l.- 25%, m.- 4%. How are the specified changes classified?

Нейтрофілія з регенеративним зсувом вліво Neutrophilia with a regenerative shift to the left

Лейкемоїдна реакція за нейтрофільним типом Leukemoid reaction by neutrophilic type

Нейтрофілія з гіперрегенеративним зсувом вліво Neuprophilia with hyperregenerative shift to the left

Нейтрофілія з дегенеративним зсувом вліво Neutrophilia with a degenerative shift to the left

Нейтрофілія з зсувом вправо Neutrophilia with shift to the right

146 / 200
Після травми хворий не може розігнути руку в ліктьовому суглобі. Порушення функції якого з основних м’язів може це спричинити? After an injury, the patient cannot extend his arm at the elbow joint. Which of the main muscles can cause this?

m. subscapularis m. subscapularis

m. triceps brachii m. triceps brachii

m. teres major m. teres major

m. infraspinatus m. infraspinatus

m. levator scapulae m. levator scapulae

147 / 200
До лікаря звернувся студент з проханням призначити препарат для лікування алергічного риніту, який виник у нього під час цвітіння липи. Який засіб можна застосувати? A student turned to the doctor with a request to prescribe a drug for the treatment of allergic rhinitis, which occurred during the flowering of the linden tree. What drug can be used?

Анаприлін Anaprilin

Лозартан Losartan

Амброксол Ambroxol

Норадреналіну гідротартрат Noradrenaline hydrotartrate

Лоратадин Loratadine

148 / 200
У зародка порушено процес сегментації дорзальної мезодерми та утворення сомітів. В якій частині шкіри можливі порушення розвитку? In the embryo, the process of segmentation of the dorsal mesoderm and the formation of somites is disturbed. In which part of the skin are developmental disorders possible?

Сальні залози Sebaceous glands

Потові залози Sweat glands

Волосся Hair

Дерма Derma

Епідерміс Epidermis

149 / 200
Дитина 9-ми місяців харчується штучними сумішами, які не збалансовані за вмістом вітаміну В6 . У дитини спостерігається пелагроподібний дерматит, судоми, анемія. Розвиток судом може бути пов’язаний з порушенням утворення: A 9-month-old child is fed with artificial mixtures that are not balanced in terms of vitamin B6 content. The child has pellagra-like dermatitis, convulsions, anemia. The development of convulsions can be due to associated with a violation of formation:

Серотоніну Serotonin

ДОФА DOFA

ГАМК GABA

Дофаміну Dopamine

Гістаміну Histamine

150 / 200
В експерименті на жабі вивчали міотатичний рефлекс. Однак при розтяганні скелетного м’яза він рефлекторно не скоротився. Порушення функції яких рецепторів може бути причиною цього? In an experiment on a frog, the myotatic reflex was studied. However, when the skeletal muscle was stretched, it did not reflexively contract. A malfunction of which receptors could be the reason for this?

Сухожильні рецептори Гольджі Golgi tendon receptors

Дотикові Touch

Больові Pain

Суглобові Articular

М’язові веретена Muscle spindles

151 / 200
В експерименті подразнюють гілочки блукаючого нерва, які іннервують серце. Це призвело до того, що припинилося проведення збудження від передсердь до шлуночків. Електрофізіологічні зміни в яких структурах серця є причиною цього? In the experiment, the branches of the vagus nerve, which innervate the heart, are irritated. This led to the fact that the conduction of excitation from the atria to the ventricles stopped. Electrophysiological changes in which structures of the heart are the cause this?

Синоатріальний вузол Sinoatrial node

Передсердя Atrial

Шлуночки Ventricles

Атріовентрикулярний вузол Atrioventricular node

Пучок Гіса His Bundle

152 / 200
Чоловік протягом 3-х років працював в одній із африканських країн. Через місяць після переїзду до України звернувся до офтальмолога зі скаргами на біль в очах, набряки повік, сльозоточивість і тимчасове послаблення зору. Під кон’юнктивою ока були виявлені гельмінти розмірами 30-50 мм, які мали видовжене ниткоподібне тіло. Який найбільш вірогідний діагноз? The man worked in one of the African countries for 3 years. A month after moving to Ukraine, he turned to an ophthalmologist with complaints of eye pain, swelling of the eyelids, lacrimation and temporary weakening of vision. Helminths measuring 30-50 mm were found under the conjunctiva of the eye, which had an elongated thread-like body. What is the most likely diagnosis?

Філяріоз Filariasis

Дифілоботріоз Diphyllobotriosis

Аскаридоз Ascariasis

Трихоцефальоз Trichocephalus

Ентеробіоз Enterobiosis

153 / 200
У юнака 16-ти років після перенесеного захворювання знижена функція синтезу білків у печінці внаслідок нестачі вітаміну K. Це може призвести до порушення: A 16-year-old boy has a reduced function of protein synthesis in the liver due to a lack of vitamin K after an illness. This can lead to a violation:

Утворення антикоагулянтів Formation of anticoagulants

Утворення еритропоетинів Formation of erythropoietins

Зсідання крові Blood clotting

Осмотичного тиску крові Blood osmotic pressure

Швидкості осідання еритроцитів Erythrocyte sedimentation rates

154 / 200
На перехід із горизонтального положення у вертикальне система кровообігу відповідає розвитком рефлекторної пресорної реакції. Що з наведеного є її обов’язковим компонентом? The circulatory system responds to the transition from a horizontal position to a vertical one with the development of a reflex pressor reaction. Which of the following is its mandatory component?

Зменшення об’єму циркулюючої крові Decreasing the volume of circulating blood

Зменшення частоти серцевих скорочень Decreasing heart rate

Зменшення насосної функції серця Decreased pumping function of the heart

Системне звуження венозних судин ємності Systemic narrowing of the venous vessels of the vessel

Системне розширення артеріальних судин опору Systemic expansion of resistance arterial vessels

155 / 200
У життєвому циклі клітини відбувається процес самоподвоєння ДНК. В результаті цього однохроматидні хромосоми стають двохроматидними. У який період клітинного циклу спостерігається це явище? In the life cycle of a cell, the process of DNA self-replication takes place. As a result, single-chromatid chromosomes become double-chromatid. In what period of the cell cycle is this phenomenon observed?

S S

G2 G2

G0 G0

M M

G1 G1

156 / 200
У водія, який потрапив у ДТП, отримав травму та знаходиться у стані шоку, спостерігається зменшення добової кількості сечі до 300 мл. Який основний патогенетичний фактор цієї зміни діурезу? A driver who got into a road accident, was injured and is in a state of shock, has a decrease in the daily amount of urine to 300 ml. What is the main pathogenetic factor of this change in diuresis?

Зменшення кількості функціонуючих клубочків Decreasing the number of functioning glomeruli

Зниження онкотичного тиску крові Reduction of oncotic blood pressure

Підвищення проникності судин Increased vascular permeability

Падіння артеріального тиску Drop in blood pressure

Вторинний гіперальдостеронізм Secondary hyperaldosteronism

157 / 200
У хворого виявлена аутоімунна гемолітична анемія, що розвивається за цитотоксичним типом. Які речовини є антигенами при алергічних реакціях II типу? The patient was diagnosed with autoimmune hemolytic anemia, which develops according to the cytotoxic type. What substances are antigens in allergic reactions of type II?

Гормони Hormones

Антибіотики Antibiotics

Модулятори запалення Inflammation modulators

Модифіковані рецептори клітинних мембран Modified cell membrane receptors

Сироваткові білки Whey proteins

158 / 200
Пацієнт страждає на геморагічний синдром, що проявляється частими носовими кровотечами, посттравматичними та спонтанними внутрішньо-шкірними та внутрішньосуглобовими крововиливами. Після лабораторного обстеження було діагностовано гемофілію В. Дефіцит якого фактора згортання крові обумовлює дане захворювання? The patient suffers from a hemorrhagic syndrome manifested by frequent nosebleeds, post-traumatic and spontaneous intra-cutaneous and intra-articular hemorrhages. After a laboratory examination, hemophilia B was diagnosed. Deficiency of which factor does blood coagulation cause this disease?

VIII VIII

VII VII

IX IX

V V

XI XI

159 / 200
Після черепно-мозкової травми у хворого спостерігається втрата можливості виконувати знайомі до травми складно-координовані рухи (апраксія). В якій ділянці кори великих півкуль найімовірніше локалізується ушкодження? After a craniocerebral injury, the patient has lost the ability to perform complex and coordinated movements familiar to the injury (apraxia). In which part of the cortex of the large hemispheres is the damage most likely localized?

Gyrus lingualis Gyrus lingualis

Gyrus paracentralis Gyrus paracentralis

Gyrus parahippocampalis Gyrus parahippocampalis

Gyrus angularis Gyrus angularis

Gyrus supramarginalis Gyrus supramarginalis

160 / 200
Чоловік 58-ми років хворіє на атеросклероз судин головного мозку. При обстеженні виявлена гіперліпідемія. Вміст якого класу ліпопротеїдів у сироватці крові даного чоловіка найбільш вірогідно буде підвищений? A 58-year-old man is suffering from atherosclerosis of cerebral vessels. During the examination, hyperlipidemia was detected. The content of which class of lipoproteins in the blood serum of this man is most likely to be increased?

Хіломікрони Chylomicrons

Ліпопротеїди низької щільності Low-density lipoprotein

Холестерин Cholesterol

Комплекси жирних кислот з альбумінами Complexes of fatty acids with albumins

Ліпопротеїди високої щільності High-density lipoprotein

161 / 200
Хворий звернувся до лікаря зі скаргами на ригідність м’язів, скутість рухів, постійний тремор рук. Встановлено діагноз: хвороба Паркінсона. Який препарат найбільш раціонально призначити? The patient turned to the doctor with complaints of muscle stiffness, stiffness of movements, constant hand tremors. The diagnosis was established: Parkinson's disease. What drug is the most rational to prescribe?

Етосуксимід Ethosuximide

Дифенін Difenin

Фенобарбітал Phenobarbital

Сибазон Sibazone

Леводопа Levodopa

162 / 200
До кардіологічного відділення надійшов хворий на ішемічну хворобу серця. Для профілактики нападів стенокардії призначено лікарський засіб з групи βадреноблокаторів. Назвіть цей препарат: A patient with ischemic heart disease was admitted to the cardiology department. To prevent angina attacks, a drug from the group of β-blockers is prescribed. Name this drug:

Морфіну гідрохлорид Morphine hydrochloride

Окситоцин Oxytocin

Метопролол Metoprolol

Атропіну сульфат Atropine sulfate

Фуросемід Furosemide

163 / 200
До гінеколога звернулася жінка 28-ми років з приводу безпліддя. При обстеженні знайдено: недорозвинені яєчники та матка, нерегулярний менструальний цикл. При досліджєнні статевого хроматину в 6ільшості соматичних клітин виявлено 2 тільця Бара. Яка хромосомна хвороба найбільш вірогідна у жінки? A 28-year-old woman consulted a gynecologist about infertility. The examination revealed: underdeveloped ovaries and uterus, irregular menstrual cycle. When examining the sex chromatin in the majority of somatic cells 2 Bar bodies were detected. What chromosomal disease is most likely in a woman?

Синдром Патау Patau syndrome

Синдром Шерешевського-Тернера Shereshevsky-Turner syndrome

Синдром трипло-Х Triple X Syndrome

Синдром Едвардса Edwards Syndrome

Синдром Клайнфельтера Klinefelter syndrome

164 / 200
При отруєнні невідомим препаратом у пацієнта спостерігались сухість слизової оболонки рота та розширення зіниць. З яким впливом пов’язана дія цього препарату? When poisoned by an unknown drug, the patient experienced dryness of the mucous membrane of the mouth and dilation of the pupils. What is the effect of this drug?

Стимуляція H-холінорецепторів Stimulation of H-cholinergic receptors

Блокада M-холінорецепторів Blockade of M-cholinergic receptors

Блокада адренорецепторів Adrenoceptor blockade

Стимуляція адренорецепторів Stimulation of adrenoceptors

Стимуляція M-холінорецепторів Stimulation of M-cholinergic receptors

165 / 200
У новонародженого спостерігається диспепсія після годування молоком. При заміні молока розчином глюкози симптоми диспепсії зникають. Недостатня активність якого ферменту спостерігається у новонародженого? A newborn has dyspepsia after feeding with milk. When replacing milk with a glucose solution, symptoms of dyspepsia disappear. Insufficient activity of which enzyme is observed in a newborn?

Амілаза Amylase

Лактаза Lactase

Ізомальтаза Isomaltase

Мальтаза Maltase

Сахараза Sucrase

166 / 200
У людини збільшена частота серцевих скорочень, розширені зіниці, сухість у роті. Наслідком активації в організмі якої системи регуляції функцій це викликано? A person has an increased heart rate, dilated pupils, dry mouth. As a result of the activation of which function regulation system in the body, this is caused?

Метасимпатична Metasympathetic

Ваго-інсулярна Vago-insular

Парасимпатична Parasympathetic

Гипоталамо-гіпофізарно-наднирникова Hypothalamic-pituitary-adrenal

Симпатична Cute

167 / 200
Хворому з переломом кінцівки необхідно призначити препарат з групи міорелаксантів деполяризуючого типу дії для проведення нетривалого хірургічного втручання. Що це за засіб? A patient with a fractured limb should be prescribed a drug from the group of muscle relaxants of the depolarizing type of action for a short surgical intervention. What kind of drug is this?

Цитітон Cititon

Пентамін Pentamine

Тубокурарину хлорид Tubocurarine chloride

Атропіну сульфат Atropine sulfate

Дитилін Ditylin

168 / 200
Пацієнт, що хворіє на хронічний бронхіт, приймає синтетичний муколітичний препарат, який сприяє розріджуванню харкотиння. Назвіть цей препарат: A patient with chronic bronchitis is taking a synthetic mucolytic drug that helps thin sputum. Name this drug:

Еналаприл Enalapril

Гепарин Heparin

Фуросемід Furosemide

Ацетилцистеїн Acetylcysteine

Діазепам Diazepam

169 / 200
У хворого з масивними опіками розвинулась гостра недостатність нирок, що характеризується значним і швидким зменшенням швидкості клубочкової фільтрації. Який механізм її розвитку? A patient with massive burns developed acute kidney failure, which is characterized by a significant and rapid decrease in the rate of glomerular filtration. What is the mechanism of its development?

Ушкодження клубочкового фільтра Damage of the glomerular filter

Збільшення тиску канальцевої рідини Increase in tubular fluid pressure

Зменшення кількості функціонуючих нефронів Decreasing the number of functioning nephrons

Зменшення ниркового кровотоку Decreased renal blood flow

Емболія ниркової артерії Renal artery embolism

170 / 200
У хворого з важким перебігом респіраторної вірусної інфекції з’явилися клінічні ознаки прогресуючої серцевої недостатності, яка призвела до смерті хворого на 2-му тижні захворювання. На аутопсії: серце зі значним розширенням порожнин, в’яле. Гістологічно в міокарді виявляється повнокров’я мікросудин і дифузна інфільтрація строми лімфоцитами та гістіоцитами. Який найбільш вірогідний діагноз? A patient with a severe respiratory viral infection developed clinical signs of progressive heart failure, which led to the death of the patient in the 2nd week of the disease. At autopsy: heart with a significant expansion of the cavities, flaccid. Histologically, the myocardium reveals full blood of microvessels and diffuse infiltration of the stroma by lymphocytes and histiocytes. What is the most likely diagnosis?

Кардіоміопатія Cardiomyopathy

Міокардит Myocarditis

Стенокардія Angina

Інфаркт міокарда Myocardial infarction

Гостра коронарна недостатність Acute coronary insufficiency

171 / 200
При поточному контролі санітарно-епідемічного стану аптеки проведено бактеріологічне дослідження повітря. Встановлено наявність у ньому бацил, дріжджеподібних грибів, гемолітичних стрептококів, мікрококів. Які з виявлених мікроорганізмів свідчать про пряму епідемічну небезпеку? During the current monitoring of the sanitary-epidemic state of the pharmacy, a bacteriological study of the air was carried out. The presence of bacilli, yeast-like fungi, hemolytic streptococci, micrococci was found in it. Which of the detected microorganisms indicate direct epidemic danger?

Бацили Bacilli

Дріжджеподібні гриби Yeast-like fungi

- -

Гемолітичні стрептококи Hemolytic streptococci

Мікрококи Micrococci

172 / 200
До реанімаційного відділення надійшов чоловік з пораненням задньої ділянки шиї (regio nuchae). Який з м’язів тіла займає цю ділянку? A man came to the intensive care unit with an injury to the back of the neck (regio nuchae). Which muscle of the body occupies this area?

m. latissimus dorsi m. latissimus dorsi

m. sternocleidomastoideus m. sternocleidomastoid

m. scalenus anterior m. scalenus anterior

m. trapezius m. trapezius

m. rhomboideus minor m. rhomboideus minor

173 / 200
На секції в лівій легені виявлено ділянку щільної тканини червоного кольору. Ділянка має форму конуса, чітко відмежована від здорової тканини, основою обернена до плеври. Тканина на розрізі зерниста, темно-червона. Який найбільш вірогідний діагноз? On the section in the left lung, an area of dense red tissue was found. The area has the shape of a cone, clearly separated from healthy tissue, the base is turned towards the pleura. The tissue on the section is granular, dark red. What is the most likely diagnosis?

Гангрена легені Gangrene of the lung

Первинний туберкульозний афект Primary tuberculosis affect

Крупозна пневмонія Croup pneumonia

Абсцес легені Lung abscess

Геморагічний інфаркт Hemorrhagic heart attack

174 / 200
У тварини збільшений тонус м’язів-розгиначів. Це є наслідком посиленої передачі інформації до мотонейронів спинного мозку такими низхідними шляхами: The animal has an increased tone of the extensor muscles. This is a consequence of the increased transmission of information to the motoneurons of the spinal cord by the following descending pathways:

Ретикулоспінальні Reticulospinal

Латеральні кортикоспінальні Lateral corticospinal

Руброспінальні Rubrospinal

Вестибулоспінальні Vestibulospinal

Медіальні кортикоспінальні Medial corticospinal

175 / 200
Після опромінювання у людини з’явилася велика кількість мутантних клітин. Через деякий час більшість із них були розпізнані і знищені клітинами імунної системи, а саме: After irradiation, a large number of mutant cells appeared in a person. After some time, most of them were recognized and destroyed by cells of the immune system, namely:

В-лімфоцитами B-lymphocytes

T-лімфоцитами-супресорами by suppressor T-lymphocytes

Стовбуровими клітинами By stem cells

T-лімфоцитами-кілерами killer T-lymphocytes

Плазмобластами Plasmoblasts

176 / 200
У чоловіка 65-ти років розвинувся гнійний абсцес на шиї. Виділена культура грампозитивних коків, яка має плазмокоагулазну активність. Більш за все, це: A 65-year-old man developed a purulent abscess on the neck. A gram-positive cocci culture was isolated, which has plasmacoagulase activity. Most of all, this:

Staphylococcus saprophyticus Staphylococcus saprophyticus

Staphylococcus epidermidis Staphylococcus epidermidis

Staphylococcus aureus Staphylococcus aureus

Streptococcus pyogenes Streptococcus pyogenes

- -

177 / 200
У клінічній практиці для лікування туберкульозу застосовують препарат ізоніазид - антивітамін, який здатний проникати у туберкульозну паличку. Туберкулостатичний ефект обумовлений порушенням процесів реплікації, окисно-відновних реакцій, завдяки утворенню несправжнього коферменту з: In clinical practice, the drug isoniazid is used for the treatment of tuberculosis - an antivitamin that is able to penetrate into the tubercle bacillus. The tuberculostatic effect is due to the violation of replication processes, redox reactions, due to the formation of false coenzyme with:

ТДФ TDF

ФАД FAD

НАД OVER

КоQ CoQ

ФМН FMN

178 / 200
У новонародженої дитини спостерігаються зниження інтенсивності смоктання, часте блювання, гіпотонія. У сечі та крові значно підвищена концентрація цитруліну. Який метаболічний процес порушений? A newborn baby has decreased sucking intensity, frequent vomiting, hypotonia. The concentration of citrulline is significantly increased in the urine and blood. What metabolic process is disturbed?

Орнітиновий цикл Ornithine cycle

Глюконеогенез Gluconeogenesis

Цикл Корі Cory Cycle

Гліколіз Glycolysis

ЦТК CTC

179 / 200
В експерименті на тварині здійснили перерізку блукаючих нервів з обох боків. Як при цьому зміниться характер дихання? In an experiment, the vagus nerves were cut on both sides of the animal. How will the nature of breathing change?

Стане глибоким і рідким It will become deep and liquid

Стане поверхневим та частим Will become superficial and frequent

Дихання не зміниться Breathing will not change

Стане глибоким і частим Will become deep and frequent

Стане поверхневим та рідким Will become shallow and liquid

180 / 200
При підготовці до видалення зуба стоматолог порекомендував пацієнтові з метою профілактики кровотечі приймати препарат. Який засіб був рекомендований? When preparing for tooth extraction, the dentist recommended the patient to take a drug to prevent bleeding. What drug was recommended?

Димедрол Diphenhydramine

Гепарин Heparin

Аспаркам Asparkam

Вікасол Vikasol

Магнію сульфат Magnesium sulfate

181 / 200
Людина, що тривалий час приймає ліки, не може різко припинити їх вживання, оскільки при цьому виникають порушення психічних та соматичних функцій. Як називається синдром різних порушень при відмові від прийому речовини? A person who has been taking drugs for a long time cannot abruptly stop their use, because mental and somatic function disorders occur. What is the name of the syndrome of various disorders when refusing taking the substance?

Абстиненція Abstinence

ідіосинкразія idiosyncrasy

Сенсибілізація Sensitization

Кумуляція Cumulative

Тахіфілаксія Tachyphylaxis

182 / 200
У хворої 45-ти років невроз, що проявляється дратівливістю, безсонням, немотивованою тривогою. Який лікарський засіб усуне всі симптоми? A 45-year-old patient has a neurosis manifested by irritability, insomnia, and unmotivated anxiety. What medicine will eliminate all symptoms?

Діазепам Diazepam

Екстракт валеріани Valerian extract

Леводопа Levodopa

Пірацетам Piracetam

Кофеїн-бензоат натрію Caffeine sodium benzoate

183 / 200
У хворого після вживання недоброякісної їжі розвинувся багаторазовий пронос. На наступний день у нього знизився артеріальний тиск, з’явились тахікардія, екстрасистолія. pH крові складає 7,18. Ці порушення є наслідком розвитку: The patient developed repeated diarrhea after consuming poor-quality food. The next day, his blood pressure dropped, tachycardia, extrasystole appeared. Blood pH is 7.18. These violations are a consequence of development:

Газового алкалозу Gas alkalosis

Газового ацидозу Gas acidosis

Негазового ацидозу Nongaseous acidosis

Метаболічного алкалозу Metabolic alkalosis

Негазового алкалозу Nongaseous alkalosis

184 / 200
Яким буде скорочення м’язів верхньої кінцівки при утриманні (але не переміщенні) вантажу в певному положенні? What will be the contraction of the muscles of the upper limb when holding (but not moving) the load in a certain position?

Ауксотонічним Auxotonic

Концентричним Concentric

Ізотонічним Isotonic

Ізометричним Isometric

Ексцентричним Eccentric

185 / 200
Хворий не відчуває дотику до шкіри в ділянці присередньої поверхні плеча. Порушення функції якого нерва спостерігається у хворого? The patient does not feel touch to the skin in the area of the medial surface of the shoulder. What nerve dysfunction is observed in the patient?

Пахвовий нерв Axillary nerve

Ліктьовий нерв Ulnar nerve

Шкірний присередній нерв плеча Cutaneous median nerve of shoulder

Шкірний присередній нерв передпліччя Cutaneous median nerve of the forearm

Променевий нерв Radiant nerve

186 / 200
По ходу слухового нерва у молодої жінки виявлена пухлина у вигляді вузла до 3 см в діаметрі, м’яко-еластичної консистенції, рожево-білого кольору, однорідна. Мікроскопічно пухлина містить пучки клітин з овальними ядрами. Клітинно-волокнисті пучки формують ритмічні структури, створені паралельними рядами, правильно орієнтованими клітинами, розташованими у вигляді частоколу, поміж яких знаходиться безклітинна гомогенна зона (тільця Верокаї). Що це за пухлина? A tumor in the form of a nodule up to 3 cm in diameter, soft-elastic consistency, pink-white in color, homogeneous, was detected along the course of the auditory nerve in a young woman. Microscopically the tumor contains bundles of cells with oval nuclei. Cellular-fibrous bundles form rhythmic structures created by parallel rows of correctly oriented cells located in the form of a picket fence, between which there is a cell-free homogeneous zone (Veroka's bodies). What kind of tumor is this?

Злоякісна невринома Malignant neuroma

Гангліоневрома Ganglioneuroma

Нейробластома Neuroblastoma

Гангліонейробластома Ganglioneuroblastoma

Невринома Neurinoma

187 / 200
До кардіологічного відділення надійшов хворий з інфарктом міокарда. Для усунення болю було вирішено потенціювати дію фентаніла нейролептиком. Який з перерахованих нейролептиків найбільш придатний для проведення нейролептаналгезії? A patient with a myocardial infarction was admitted to the cardiology department. To eliminate pain, it was decided to potentiate the effect of fentanyl with a neuroleptic. Which of the listed neuroleptics is most suitable for neuroleptanalgesia?

Галоперидол Haloperidol

Трифтазін Triftazine

Сульпірид Sulpiride

Аміназин Aminazine

Дроперидол Droperidol

188 / 200
Для лікування бактеріальної пневмонії було призначено бензилпеніциліну натрієву сіль. Який механізм антимікробної дії препарату? Benzylpenicillin sodium salt was prescribed for the treatment of bacterial pneumonia. What is the mechanism of antimicrobial action of the drug?

Пригнічення внутрішньоклітинного синтезу білка Inhibition of intracellular protein synthesis

Антагонізм з параамінобензойною кислотою Antagonism with para-aminobenzoic acid

Пригнічення синтезу клітинної стінки мікроорганізмів Suppression of the synthesis of the cell wall of microorganisms

Пригнічення активності холінестерази Inhibition of cholinesterase activity

Пригнічення SH-груп ферментів мікроорганізмів Suppression of SH-groups of enzymes of microorganisms

189 / 200
Хворий 49-ти років, водій за професією, скаржиться на нестерпний стискаючий біль за грудниною, що 'віддає' у ділянку шиї. Біль виник 2 години тому. Об’єктивно: стан важкий, блідість, тони серця послаблені. Лабораторне обстеження показало високу активність креатинкінази та ЛДГ1 . Для якого захворювання характерні такі симптоми? A 49-year-old patient, a driver by profession, complains of unbearable squeezing pain behind the sternum, which 'gives' to the neck. The pain started 2 hours ago. 'objectively: the condition is severe, pallor, heart sounds are weakened. Laboratory examination showed high activity of creatine kinase and LDH1. What disease is characterized by such symptoms?

Стенокардія Angina

Гострий панкреатит Acute pancreatitis

Жовчнокам’яна хвороба Cholelithiasis

Цукровий діабет Diabetes

Гострий інфаркт міокарда Acute myocardial infarction

190 / 200
У чоловіка 60-ти років спостерігається послаблення перистальтики кишечнику. Який з наведених харчових продуктів буде стимулювати перистальтику найбільше? A 60-year-old man has decreased intestinal peristalsis. Which of the following food products will stimulate peristalsis the most?

Сало Lard

Чай Tea

Білий хліб White bread

М’ясо Meat

Чорний хліб Black bread

191 / 200
При посіві матеріалу із зіву від хворого ангіною на кров’яно-телуритовий агар виросли колонії діаметром 4-5 мм, сірого кольору, радіально посмуговані (у вигляді розеток). Під мікроскопом - грампозитивні палички із булавоподібними потовщеннями на кінцях, що розміщені у вигляді розчепірюваних пальців. Які це мікроорганізми? When inoculating the material from the pharynx of a patient with angina on blood-tellurite agar, colonies with a diameter of 4-5 mm, gray color, radially striated (in the form of rosettes) grew . Under the microscope - gram-positive rods with club-like thickenings at the ends, arranged in the form of splayed fingers. What are these microorganisms?

Дифтероїди Diphtheroids

Стрептококи Streptococci

Клостридії ботулізму Clostridia botulism

Стрептобацили Streptobacilli

Коринебактерії дифтерії Diphtheria corynebacteria

192 / 200
На мікропрепараті підщелепної слинної залози навколо кінцевих відділів і вивідних проток розрізняються кошикоподібні клітини, які охоплюють основи сероцитів і називаються міоепітеліоцити. До якої тканини належать ці клітини? On a microscopic specimen of the submandibular salivary gland, basket-like cells can be seen around the end sections and excretory ducts, which cover the bases of serocytes and are called myoepitheliocytes. What tissue do these cells belong to?

Нервова Nervous

Пухка волокниста сполучна Loose fibrous binder

Сполучна зі спеціальними властивостями Connected with special properties

Епітеліальна Epithelial

М’язова Muscular

193 / 200
Встановлено, що аглютинація еритроцитів крові реципієнта викликали стандартні сироватки I та II груп і не викликали - сироватка III групи і антирезусна сироватка. Кров якої групи за системами AB0 і резус можна переливати реципієнту? It was established that agglutination of erythrocytes of the recipient's blood was caused by standard serums of groups I and II and not by serum of group III and anti-Rhesus serum. Blood of which group according to the AB0 and Rhesus systems can it be transferred to the recipient?

AB(IV), Rh- AB(IV), Rh-

AB(IV), Rh+ AB(IV), Rh+

0,α,β, (I)Rh+ 0,α,β, (I)Rh+

B,α (III)Rh- B,α (III)Rh-

194 / 200
Фармакологічні ефекти антидепресантів пов’язані з блокуванням (інгібуванням) ними ферменту, який каталізує розпад таких біогенних амінів, як норадреналін і серотонін в мітохондріях нейронів головного мозку. Який фермент бере участь у цьому процесі? The pharmacological effects of antidepressants are associated with their blocking (inhibition) of an enzyme that catalyzes the breakdown of such biogenic amines as norepinephrine and serotonin in the mitochondria of brain neurons. What enzyme participates in this process?

Пептидаза Peptidase

Моноамінооксидаза Monoamine oxidase

Трансаміназа Transaminase

Декарбоксилаза Decarboxylase

Ліаза Liaz

195 / 200
Онкологічному хворому призначили препарат метотрексат, до якого з часом клітинимішені пухлини втратили чутливість. Експресія гену якого ферменту при цьому змінюється? A cancer patient was prescribed the drug methotrexate, to which over time the target tumor cells lost sensitivity. The gene expression of which enzyme changes?

Дезаміназа Deaminase

Фолатоксидаза Folate oxidase

Дегідрофолатредуктаза Dehydrofolate reductase

Фолатдекарбоксилаза Folate decarboxylase

Тиміназа Thyminase

196 / 200
До кардіологічного відділення надійшов хворий з гіпертонічним кризом, йому внутрішньовенно ввели антигіпертензивний засіб - сіль лужноземельного металу. Який препарат ввели хворому? A patient with a hypertensive crisis was admitted to the cardiology department, an antihypertensive drug was administered intravenously - an alkaline earth metal salt. What drug was administered to the patient?

Калію хлорид Potassium chloride

Бензогексоній Benzohexonium

Магнію сульфат Magnesium sulfate

Натрію гідрокарбонат Sodium bicarbonate

Кальцію лактат Calcium lactate

197 / 200
Постраждалий доставлений до хірургічного відділення із проникним пораненням у лівій бічній ділянці живота. Який відділ товстої кишки, найімовірніше, ушкоджений? The victim was brought to the surgical department with a penetrating wound in the left side of the abdomen. Which part of the colon is most likely injured?

Colon descendens Colon descendens

Colon transverses Colon transverses

Colon ascendens Colon ascendens

Caecum Caecum

Rectum Rectum

198 / 200
Постраждалого в аварії водія госпіталізовано до стаціонару з ушкодженням медіального надвиростка плечової кістки. Який нерв при цьому може бути ушкоджений? The driver injured in the accident was hospitalized with an injury to the medial epicondyle of the humerus. Which nerve can be damaged?

n. ulnaris n. ulnaris

n. muscolocutaneus n. muscolocutaneus

n. radialis n. radialis

n. axillaris n. axillaris

n. medianus n. medianus

199 / 200
Чоловік з колотою раною в ділянці чотирьохстороннього отвору звернувся до лікаря. При обстеженні виявлено, що потерпший не може відвести руку від тулуба. Який нерв вірогідно ушкоджений? A man with a stab wound in the area of the quadrilateral opening went to the doctor. During the examination, it was found that the victim could not move his arm away from the body. Which nerve is probably damaged?

n. subclavius n. subclavius

n. ulnaris n. ulnaris

n. medianus n. medianus

n. axillaris n. axillaris

n. radialis n. radialis

200 / 200
У вагітної жінки на передній черевній стінці виявлено пухлиноподібне утворення, яке виникло на місці видаленої два роки тому пухлини. Утворення має щільну консистенцію і розміри 2х1 см, з чіткими межами. При гістологічному дослідженні виявлено, що пухлина побудована з диференційованої сполучної тканини з переважанням колагенових волокон. Про яку пухлину слід думати? A tumor-like formation was detected on the front abdominal wall of a pregnant woman, which arose at the site of a tumor removed two years ago. The formation has a dense consistency and dimensions of 2x1 cm, with clear borders . During histological examination, it was found that the tumor is made of differentiated connective tissue with a predominance of collagen fibers. What kind of tumor should we think about?

Гібернома Hibernoma

Фібросаркома Fibrosarcoma

Лейоміома Leiomyoma

Ліпома Lipoma

Десмоїд Desmoid